Gram-Positive Bacteria MCQ

Ace your homework & exams now with Quizwiz!

*The answer is A.* Hematogenous osteomyelitis is predominantly a disease of children that most frequently affects the long bones. When this condition occurs in adults, it is frequently due to a predisposition for bacteremia such as IV drug abuse or dialysis. The presenting symptoms of hematogenous osteomyelitis are vague, and an increased clinical suspicion is required to arrive at the diagnosis. Commonly, symptoms are similar to those of any systemic infection: malaise, fevers and pain over one of the long bones. Bone pain is caused by abscess formation within the bone leading to periosteal disruption and bone necrosis. Staphylococcus aureus is implicated in most cases of acute hematogenous osteomyelitis in otherwise healthychildren.

A 10-year-old Caucasian male is brought to the ER with a several day history of high fevers and chills. He also complains of dull pain just above the left knee. There is no joint effusion. X-ray films show soft tissue swelling, bone destruction, and periosteal reaction over the lower end of femur. Which of the following organisms is most likely responsible for this patient's symptoms? A. Staphylococcus aureus B. Staphylococcus epidermidis C. Streptococcus pyogenes D. Streptococcus agalactiae E. Streptococcus pneumoniae F. Streptococcus faecalis G. Moraxella catarrhalis

*The answer is A.* Listeria. monocytogenes grows optimally at 30 to 37°C, but is capable of growth at 4°C. Thus, refrigeration does not reliably suppress its growth in food. L. monocytogenes is a catalase-positive, gram-positive, obligate intracellular pathogen. These organisms are found in cattle, other warm-blooded animals, and fish, where they can cause disease.

Listeria monocytogenes shows which of the following characteristics? A. It can grow at refrigerator temperatures (4°C). B. It is an extracellular pathogen. C. It is catalase negative. D. It is a gram-negative coccus. E. It is strictly a human pathogen.

*The answer is C.* The polysaccharide capsule of Streptococcus pneumoniae prevents phagocytosis, or ingestion.

A 35-year-old woman delivers a newborn who develops meningitis. The mother has a streptococcus isolated from her vagina. The organism agglutinates with antiserum directed against type B surface carbohydrate. The virulence of this organism is related to a bacterial constituent that interferes with which of the following host phagocyte functions? (A) Aggregation (B) Chemotaxis (C) Ingestion (D) Intracellular killing (E) Pseudopod formation

*The answer is C.* Humans are a natural reservoir for S. aureus. Very few strains of S. aureus are susceptible to penicillin, and more than 50% have become methicillin resistant (MRSA). MRSA can be hospital acquired or community acquired. Asymptomatic colonization of the nasopharynx is most common, and this is the site sampled by epidemiologists wishing to study S. aureus carriage. A colonized individual can be a carrier and can transmit the organism by direct contact. In the general population, 25-30% of individuals have nasal colonization with Staphylococcus aureus. Nasal carriage increases the risk of infections following surgery, peritoneal dialysis and hemodialysis.

A county hospital experiences an outbreak of methicillin-resistant Staphylococcus aureus (MRSA) wound infections. You suspect that many staff members are carriers of the bacteria. Which of the following would you expect to be the most commonly MRSA populated site in staff members? A. Hands B. Axilla C. Anterior nares D. Perineum E. Oropharynx

*The answer is E.* Assuming that the child has no deficiencies with humoral immunity, the standard of care is to provide a booster of tetanus toxoid to a patient with the above clinical picture. It is currently believed that the anamnestic response (i.e., memory cell activity) will produce ample antibody long before the tetanus bacillus is able to produce the damaging toxin. In other words, the patient is actively immunized.

A high school student who was appropriately immunized with the diphtheria-tetanus-acellular pertussis (DTaP) series by kindergarten, but who had received no additional boosters since, stepped on a nail while walking barefoot. The nail was easily removed and the lesion freely bled. What should the student be given? (A) Equine tetanus antitoxin (B) Human tetanus immune globulin (C) Prophylactic antibiotic treatment for Clostridium tetani (D) No treatment (E) Tetanus toxoid

*The answer is B.* Bacillus is the correct genus. There are two Gram-positive spore-forming rods. Bacillus is aerobic with one species (Bacillus anthracis), causing hemorrhagic lymphadenitis and pulmonary edema. The other genus with bacterial endospores, the Clostridia, do not grow aerobically. The other Gram-positive organism listed, Actinomyces, often described as a branching bacterium, is also anaerobic. Campylobacter and Haemophilus are both Gram-negative.

A patient presents with rapid onset severe respiratory symptoms. Chest radiographs show a hemorrhagic lymphadenitis. The isolation of chains of fairly large, aerobic Gram-positive rods, some of which have started to sporulate from a patient with this presentation, should raise a major concern of which organism? (You should be able to answer this question from the genus alone, although a question might also mention that it was nonmotile.) (A) Actinomyces israelii (B) Bacillus anthracis (C) Campylobacter jejuni (D) Clostridium perfringens (E) Haemophilus influenzae

*The answer is B.* Staphylococcus epidermidis is ubiquitous as part of the normal flora. Organisms are introduced into the host during invasive procedures. Staph. aureus is more likely to be acute, with high fever and damage developing more quickly.

A patient who had surgery to put in a pacemaker and who states he felt fine for the first 2 months now presents 3 months postoperatively with complaints of malaise and increasing fatigue. He is running a low-grade fever, tires easily, and has worsening heart murmurs. Which of the following staphylococcal organisms causes subacute bacterial endocarditis that generally occurs 2 months or more after heart surgery? (A) Staphylococcus aureus (B) Staphylococcus epidermidis (C) Staphylococcus haemolyticus (D) Staphylococcus saprophyticus

*The answer is A.* This case is a classic description of botulism. The disease, although rare, is associated with ingestion of home-canned food products. The symptoms of botulism are diffuse, flaccid paralysis, visual disturbances including diplopia (double vision), and speech difficulty (dysarthria). None of the other choices are associated with these disease manifestations.

A previously healthy woman became nauseated and vomits 12 hours after eating home-pickled eggs. Within 24 hours, she developed a diffuse flaccid paralysis and respiratory impairment, necessitating hospitalization and mechanical ventilation. Other symptoms included diplopia and dysarthria. What is the most likely diagnosis? (A) Botulism (B) Brucellosis (C) Gastrointestinal anthrax (D) Legionellosis (E) Meningococcal disease

*The answer is C.* Lecithinase is the main toxin of C. perfringens; its concentration correlates with its lethal and necrotic effects. Lecithinase, also known as phospholipase C or alpha toxin is an enzyme that catalyzes the splitting of phospholipid molecules. (Recall that human phospholipase A2 is the enzyme that catalyzes arachidonic acid release from phospholipid cell membranes in the first step of leukotriene, thromboxane, and prostaglandin synthesis.) Phospholipase C hydrolyzes lecithin-containing lipoprotein complexes in cell membranes causing cell lysis (including RBC hemolysis), tissue necrosis and edema. C. perfringens has atleast 2 toxins of which Alpha toxin is the most injurious.

A toxic substance produced by C. perfringens induces massive hemolysis and tissue necrosis when injected into mice. The lethal effect observed in the experiment correlates with the substance's ability to split A. Collagen B. Hyaluronic acid C. Phospholipids D. Carbohydrates E. Plasminogen

*The answer is A.* Staphylococcus aureus is positive for coagulase test which is a test for the ability of the bacteria to cause blood plasma to clot.

Among the following which is positive for coagulase test? a) Staphylococcus aureus b) Staphylococcus epidermidis c) Staphylococcus saprophyticus d) E.coli

*The answer is C.* Bacillus Anthracis has a poly-D-glutamic acid protein capsule that prevents phagocytosis.

Poly-D-glutamic acid protein capsule that prevents phagocytosis. A. Bacillus Cereus B. Clostridium Botulium C. Bacillus Anthracis D. Clostridium Tetani E. Listeria monotogenes

*The answer is A.* The staphylococci are aerobic while the streptococci are aerotolerant anaerobes. Streptococci ferment even in the presence of full oxygen and lack catalase. A standard quick test is the test mixing staphylococci with hydrogen peroxide. The generation of oxygen bubbles indicates that a Gram-positive coccus is a Staphylococcus rather than a Streptococcus. A coagulase test using serum to see if fibrinogen is clotted is used to distinguish the coagulase-positive staphylococci from coagulase-negative staphylococci. Oxidase is not the reagent but the reactor in the oxidase test.

The reagent used to distinguish staphylococci from streptococci is (A) Hydrogen peroxide (B) Fibronectin (C) Fibrinogen (D) Oxidase

*The answer is C.* Streptococcus pyogenes is highly sensitive to bacitracin, and diagnostic disks with a very low concentration of the antibiotic inhibit growth in culture. All streptococci are catalase negative. The growth of Streptococcus pneumoniae is inhibited by optochin. Most streptococci are aerotolerant anaerobes, and grow fermentatively even in the presence of oxygen. Enterococcus faecalis is γ hemolytic (no hemolysis).

Which of the following statements is correct? A. Streptococci are catalase positive. B. Growth of Streptococcus pneumoniae is not sensitive to optochin. C. Streptococcus pyogenes is highly sensitive to bacitracin. D. Streptococci are obligate anaerobes. E. Enterococcus faecalis is β-hemolytic.

*The answer is E.* This case is descriptive of subacute endocarditis. The Gram stain shows Gram-positive cocci in chains and the culture shows α-hemolysis. This information, along with the fact that the organism is catalase-negative, suggests that the organism is a viridans group Streptococcus. This group comprises several species. They are important causes of dental plaque and can be introduced into the blood stream where they cause subacute endocarditis. Both group A and group B streptococci are β-hemolytic. Staphylococcal species are catalase-positive.

While on vacation, a 68-year-old man develops fever, chills, night sweats, and malaise that began insidiously and progressed over several days. He sought care at an urgent care facility and was diagnosed clinically with the flu. He cuts his vacation short and returns home with the same symptoms plus weakness and dyspnea. He sees his primary care physician who hears a new onset of murmur on auscultation and admits the man to the hospital. Blood cultures grow the catalase-negative organism shown in the accompanying photographs. What is the cause of this man's infection? (A) Group A Streptococcus pyogenes (B) Group B Streptococcus agalactiae (C) Staphylococcus aureus (D) Staphylococcus epidermidis (E) Viridans group streptococci

*The answer is A.* The secretion of dextran, a sticky polysaccharide, enables the organisms to adhere to surfaces such as tooth enamel or heart valves. Hyaluronidase is produced by invasive organisms. Viridans group streptococci are not characterized as invasive. Lipopolysaccharide is a component of the outer membrane of Gram-negative bacteria. Pyrogenic exotoxins are produced by certain strains of group A Streptococci associated with scarlet fever. Exfoliative toxins are produced by certain strains of Staphylococcus aureus and are responsible for scalded skin syndrome.

While on vacation, a 68-year-old man develops fever, chills, night sweats, and malaise that began insidiously and progressed over several days. He sought care at an urgent care facility and was diagnosed clinically with the flu. He cuts his vacation short and returns home with the same symptoms plus weakness and dyspnea. He sees his primary care physician who hears a new onset of murmur on auscultation and admits the man to the hospital. Blood cultures grow the catalase-negative organism shown in the accompanying photographs. Which of the following is an important virulence factor of the organisms in the above question? (A) Dextran production (B) Hyaluronidase (C) Lipopolysaccharide (D) Pyrogenic exotoxins (E) Exfoliative toxins

*The answer is E.* Resistance is due to the production of β-lactamase by the bacteria.

A 2-year-old boy is diagnosed with staphylococcal scalded skin syndrome. In vitro studies show the causal organism to be resistant to penicillin. Which of the following mechanisms of action is most likely involved in this resistance? (A) Active efflux of the antibiotic from the bacteria (B) Decreased uptake of the antibiotic into the bacteria (C) Mutation of the 30S ribosomal subunit (D) Mutation of the 50S ribosomal subunit (E) Production of β-lactamase by the bacteria

*The answer is C.* Both Pseudomonas exotoxin A and diphtheria toxin inhibit protein synthesis through the inhibition of elongation factor (EF-2). Incorrect choices include: Shiga toxin, which is a cytotoxin, enterotoxin, and neurotoxin. Vibrio cholerae enterotoxin and E. coli labile toxin (LT) both result in increased cyclic adenosine monophosphate (cAMP).

A 22-year-old man with cystic fibrosis presents with fever and increasing dyspnea. A Gram-negative organism is found in unusually high numbers in the pulmonary mucus. From the above case, exotoxin A of the causative agent most closely resembles the action of which other microbial toxin? (A) Heat-labile toxin (LT) of Escherichia coli (B) Shiga toxin (C) Diphtheria toxin (D) Vibrio cholerae toxin (E) Verotoxin

*The answer is E.* This organism, a frequent skin colonizer, can contaminate food during preparation. The food is then left at room temperature for several hours during which time the bacteria replicate and secrete heat-stable enterotoxins. The toxins enter the blood and affect the vomiting control center of the brain, inducing vomiting 1 to 6 hours following ingestion of contaminated foods. Settings at risk for outbreak of staphylococcal food poisoning include group picnics, restaurants, and school cafeterias or other situations in which large numbers of people eat food made in batches by food handlers, colonized with the bacteria, who do not use precautions to prevent contamination. Foods that are most frequently implicated are those that are high in salt or sugar, such as salted meats, ham, potato or macaroni salad, and cream-filled pastries or pies. C. jejuni causes a diarrheal illness and is associated with the consumption of undercooked chicken. C. perfringens also causes a diarrheal illness with an incubation period of 8 to 14 hours following ingestion of contaminated food. B. cereus causes vomiting and is most frequently associated with the consumption of fried rice. Salmonella food poisoning causes a bloody diarrhea 16 hours to 2 days after ingestion of undercooked chicken, eggs, or other contaminated foods.

A 32-year-old woman presents to an urgent care facility with severe vomiting. She claims that several other people who ate at the church picnic 4 hours earlier are also sick with the same symptoms. She is clinically diagnosed with food poisoning. A health department investigation reveals that all affected individuals had eaten the ham salad. What is the bacterial agent responsible for the signs and symptoms? (A) Campylobacter jejuni (B) Clostridium perfringens (C) Bacillus cereus (D) Salmonella sp. (E) Staphylococcus aureus

*The answer is C.* The case describes the outcome of an infection with Listeria monocytogenes in a pregnant woman. L. monocytogenes is a motile, Gram-positive coccobacillus that can grow at refrigerated temperatures, a characteristic exploited for laboratory diagnosis of this pathogen. Infections with L. monocytogenes are linked with contamination of food with fecal material from cattle or other animals whose intestines are colonized by this organism. Infection of young, healthy adults is often asymptomatic or leads to a flu-like illness or diarrhea. Immune compromised individuals, pregnant women, and the elderly can expierence life-threatening bacteremia or meningitis. Fetal infection can lead to fetal death or early onset of neonatal disease characterized by widespread abscesses and granulomas in many organs. Transient intestinal colonization with L. monocytogenes can occur in humans; however, colonization does not lead to disease. The organism is not transmitted person-to-person.

A 34-year-old woman in her third trimester of pregnancy delivers a stillborn infant. Her history is significant for a febrile illness with headache, back pain, and diarrhea prior to going into premature labor. A Gram-positive motile coccobacillus was isolated from blood and tissue samples of the mother and the infant cultured at 4 °C. How did the mother acquire this infection? (A) Acquisition of a respiratory pathogen that spread hematogenously (B) Contamination of genital tract with normal intestinal flora (C) Ingestion of contaminated food (D) Overgrowth of normal vaginal flora (E) Sexual transmission

*The answer is C.* If the mother is young and has had multiple sexual partners, she is more likely to be colonized with Group B streptococci. If the labor is prolonged after rupture of the membranes, the baby is more likely to be infected. And, since she delivered before her due date and had her baby at home, she was not screened for Group B streptococci and did not receive intrapartum antibiotics to prevent infection of the baby. The other descriptions belong to: (A) Enterococcus, (B) Strep. pneumoniae, and (D) Viridans strep.

A 4-day-old infant girl now showing signs of sepsis is brought to the emergency department. She was preterm (33 weeks) and born at home to her 16-year-old mom after 22 hours of labor following the rupture of the membranes. A friend helped the mother deliver the baby. What is the best description for the agent most likely causing the sepsis if it was acquired during labor but prior to delivery? All organisms in the answer choices are Gram-positive, catalase-negative cocci found in pairs or short chains. (A) Nonhemolytic organisms found as part of the normal fecal flora; resistant to bile and optochin; carries a high level of drug resistance (B) Alpha-hemolytic diplococci sensitive to both bile and optochin (C) Beta-hemolytic cocci in chains and carrying Lancefield's Group B antigen (D) Alpha-hemolytic cocci in chains; resistant to bile and optochin

*The answer is B.* Of the answer choices, only streptococci and staphylococci are Gram-positive. The streptococci are catalase-negative and staphylococci are catalase-positive. Of the two staphylococci, Staphylococcus aureus is the beta-hemolytic, coagulase-positive organism.

A 54-year-old man develops a pyogenic infection along the suture line after knee surgery. The laboratory gives a preliminary report of a beta-hemolytic, catalase- positive, coagulase-positive, Gram-positive coccus. The most likely causative agent is (A) Moraxella catarrhalis (B) Staphylococcus aureus (C) Staphylococcus epidermidis (D) Streptococcus agalactiae (E) Streptococcus pyogenes

*The answer is C.* This case is descriptive of acute rheumatic fever. This complication of pharyngeal infection with group A Streptococci presents usually 2 to 3 weeks after recovery from sore throat, manifests as a combination of symptoms including polyarthritis, carditis, and chorea, and is uncommon in developed countries where antibiotic treatment of strep throat prevents the occurrence of rheumatic fever. Repeat episodes of rheumatic fever lead to chronic rheumatic heart disease, one of the leading causes of childhood heart diease in the developing world. Because it occurs weeks after pharyngeal infection, throat cultures are seldom positive. Detection of antibodies to streptolysin O, a pore-forming cytolysin released from group A streptococci, is used to document recent infection with this organism. Elevated C-reactive protein and elevated erythroyte sedimentation rate are indicative of an acute phase response. They are two of the "minor criteria" for the diagnosis of acute rheumatic fever using the Jones criteria. They are a nonspecific indicator of a systemic inflammatory response. Lancefield agglutination tests enable laboratory classification of a streptococcal isolate into the various Lancefield groups. This test is not used to make a diagnosis of recent streptococcal infection. The rheumatoid factor test is used as an aid in the diagnosis of rheumatoid arthritis, an autoimmune disease.

A 6-year-old boy in Pakistan presents to a local hospital with involuntary limb movements. He also complains of pain and swelling in several joints including the wrists, elbows, knees, and ankles as well as weakness and shortness of breath. About 2 weeks earlier, he had been ill with fever and sore throat. A throat culture for group A Streptococcus was negative. What other test result would indicate a recent infection with this organism? (A) Elevated C-reactive protein (B) Elevated erythrocyte sedimentation rate (C) Positive antistreptolysin O antibodies (D) Positive Lancefield agglutination test (E) Positive Rheumatoid factor test

*The answer is B.* The M protein is a dimeric, helical protein anchored in the cytoplasmic membrane and extending through the cell wall and capsule of group A streptococci (GAS). It is highly antigenically variable and is the basis for distinguishing the 90+ serotypes of GAS. The M protein of those strains associated with acute rheumatic fever (rheumatogenic strains) is antigenically similar to components of heart valves and connective tissue in joints. Thus, an immune response directed against the M proteins of rheumatogenic strains cross reacts with similar epitopes on heart valves or joint connective tissue leading to acute rheumatic fever. The capsule is an important virulence factor for GAS but does not incite the autoimmune attack that is at the heart of the pathogenesis of acute rheumatic fever. The other virulence factors listed do not play a role in this disease.

A 6-year-old boy in Pakistan presents to a local hospital with involuntary limb movements. He also complains of pain and swelling in several joints including the wrists, elbows, knees, and ankles as well as weakness and shortness of breath. About 2 weeks earlier, he had been ill with fever and sore throat. A throat culture for group A Streptococcus was negative. With regard to the above question, what is the major virulence factor of this organism that plays a role in the pathogenesis of the boy's illness? (A) Capsule (B) M protein (C) Phage-associated toxin (D) Secretion of dextran (E) Streptococcal superantigen

*The answer is A.* An increasing number of pneumococcal strains make penicillin-binding proteins that are not optimally recognized by penicillins and aminopenicillins. Pneumococci do not make β-lactamase. Growing resistance to macrolides, tetracycline, and sulfa drugs is also occurring in strains of this organism, possibly by some of the mechanisms listed.

A 74-year-old woman is hospitalized with community- acquired pneumonia. Blood and sputum cultures grew the α-hemolytic catalase-negative, bile-soluble, optochin-sensitive organism shown in the photograph. It showed an intermediate degree of resistance to penicillin. What is the molecular basis of resistance to penicillin shown by the organism in the above question? (A) Altered penicillin-binding proteins (B) Efflux pump (C) Mutation of ribosomal binding site (D) Mutated porin proteins (E) Production of β-lactamase

*The answer is A.* The organism described in this case is Streptococcus pneumoniae. It is an important cause of community-acquired pneumonia and strains range in sensitivity to penicillin from fully susceptible to intermediately susceptible to highly resistant. Virulent strains of S. pneumoniae, also called pneumococcus, produce a thick, polysaccharide capsule that provides protection against complement activation and phagocytosis. Pneumolysin is a unique cytotoxin elaborated by pneumococci. It induces pores in host cell plasma membranes following binding to cholesterol residues, thereby damaging ciliated respiratory epithelial cells and neutrophils. The organism does not elaborate any of the other virulence factors listed. Filamentous hemagglutinin, tracheal cytotoxin, and pertactin are virulence factors of Bordetella pertussis. Neuraminidase and hemagglutinin are envelope glycoproteins of influenza virus. Pyocyanin and endotoxin are two of the multiple virulence factors of Pseudomonas aeruginosa.

A 74-year-old woman is hospitalized with community- acquired pneumonia. Blood and sputum cultures grew the α-hemolytic catalase-negative, bile-soluble, optochin-sensitive organism shown in the photograph. It showed an intermediate degree of resistance to penicillin. What are the other important virulence factors of this organism? (A) Capsule and pneumolysin (B) Filamentous hemagglutinin and superantigen (C) Neuraminidase and hemagglutinin (D) Pyocyanin and endotoxin (E) Tracheal cytotoxin and pertactin

*The answer is A.* Entero cocci are important causes of nosocomial urinary tract infections (UTIs). They are found as normal intestinal flora and cause disease when introduced into the body. UTI is the most common disease manifestation of this group. Enterococci are innately resistant to cephalosporins as this group of antibiotics cannot bind to enterococcal penicillin-binding proteins. Enterococci are able to utilize folic acid in their environment and are thus insensitive to TMP-SMZ. Although ampicillin plus an aminoglycoside is often used to treat enterococcal UTIs, these organisms show intrínsic resistance to aminoglycosides alone. Vancomycin resistance is the most alarming form of resistance among strains of enterococci as it is feared that the gene for this resistance could be shared among other species of bacteria, most notably, methicillin-resistant Staphylococcus aureus. Of the other organisms listed, only S. saprophyticus is a common uropathogen; however, it is usually associated with community rather than hospital-acquired UTIs and it is not noted for significant antibiotic resistance.

A 76-year-old hospitalized male being treated successfully with vancomycin for hospital-acquired methicillin-resistant Staphylococcus aureus bacteremia develops a new onset of fever. A urinary tract infection is suspected and a Gram stain of unspun urine reveals neutrophils and Gram-positive cocci. Antibiotic sensitivity results of the organism cultured from the urine reveal resistance to cephalosporins, TMP-SMZ, aminoglycosides, and vancomycin. Which bacteria is the most likely cause of his urinary tract infection? (A) Enterococcus faecalis (B) Group B Streptococcus (C) Peptostreptococcus (D) Staphylococcus saprophyticus (E) Streptococcus pyogenes

*The answer is D.* The Gram-positive organism Streptococcus pneumoniae contains no endotoxin. It is not phagocytosed in the immunologically naive, eliminating choice B. It is the capsule that is considered the most important virulence factor.

A 78-year-old man presents with a high fever, cough producing a blood-tinged sputum, and difficulty breathing. Sputum shows an organism consistent with Streptococcus pneumoniae. What is the most important virulence factor? (A) Endotoxin (B) A phospholipase allowing Streptococcus pneumoniae to escape the phagosome quickly (C) Polypeptide capsule (D) Polysaccharide capsule

*The answer is F.* The most common cause of pneumonia in people over 65 years of age has generally been S. pneumoniae. Although the other agents also cause pneumonia, S. pneumoniae is the only organism that fits the descriptions of Gram-positive cocci in chains. It is also quite common following influenza. If the woman had not been vaccinated for influenza, then she also may not have been vaccinated for pneumococcus.

An 83-year-old who still lives in her own home has developed pneumonia following influenza. The Gram stain of her sputa is shown. What is the most likely agent? (A) Chlamydophila pneumoniae (B) Influenza virus (C) Klebsiella pneumoniae (D) Mycoplasma pneumoniae (E) Staphylococcus aureus (F) Streptococcus pneumoniae

*The answer is E.* Autoclaving ensures that spores are inactivated.

An investigator conducts an experiment on Clostridium perfringens and then sterilizes the culture dishes by autoclaving. This method of sterilization is most appropriate because it ensures that which of the following bacterial structures are inactivated? (A) Adhesion factors (B) Endotoxins (C) Fimbriae (D) Pili (E) Spores

*The answer is E.* This patient has food-poisoning from preformed enterotoxins from Staphylococcus aureus. This usually occurs within 1-8 hours of eating the contaminated food.

At a banquet, the menu includes fried chicken, home-fried potatoes, peas, chocolate eclairs, and coffee. Within 2 hours, most of the diners become violently ill, with nausea, vomiting, abdominal pain, and diarrhea. Analysis of the contaminated food is most likely to yield large numbers of which of the following organisms? (A) Enterococcus faecalis (B) Escherichia coli (C) Proteus mirabilis (D) Salmonella typhimurium (E) Staphylococcus aureus

*The answer is E.* Listeria is the correct answer. Both Bacillus and Clostridium can be eliminated because they are spore-formers. Actinomyces and corynebacteria are both Gram-positive and nonmotile and are not common causative agents of neonatal meningitis. Listeria has a tumbling motility when grown in broth, in this case CSF.

The CSF from a 2-week-old infant with meningitis shows rods with tumbling motility. These bacteria are found to be Gram-positive and do not form spores. What is the most likely agent? (A) Actinomyces (B) Bacillus (C) Clostridium (D) Corynebacterium (E) Listeria

*The answer is B.* Clostridial myonecrosis or gas gangrene is caused by a number of clostridial species, the most common being Clostridium perfringens. All are large, Gram-positive, anaerobic bacilli. When stressed, in the environment or possibly the laboratory, they will produce endospores. The soil contaminating the punctured wound was the vehicle for infection in this particular case. Infections often prove exquisitely painful. Gangrene is a fulminate condition, with severe illness and even mortalities possible within hours of the initial insult, especially if untreated. Clostridia are usually sensitive to penicillins and other β-lactam antibiotics.

While planting a tree, a man punctured his foot through his tennis shoe with the prongs of a rake. He was not initially concerned with the wound and washed it only superficially. At 4:00 AM, he awakened with excruciating pain in the foot and sought medical care at the emergency department. The lesion was cleaned and excised of devitalized tissue and an IV β-lactam antibiotic was administered, which subsequently cleared the infection. A Gram stain of an anaerobic culture taken from wound secretions is shown. Of the following microbial groups, which is most compatible with this case? (A) Actinomycetes (B) Clostridia (C) Pseudomonads (D) Staphylococci (E) Streptococci

*The answer is D.* Clostridium botulinum found in household dust or honey was ingested by the baby and the spores germinated in her GI tract because her normal flora was not sufficient to suppress the germination. It is the vegetative cells that produce the botulinum toxin. Antibiotics disrupt normal flora, prolonging the disease, but administration of human antitoxin can dramatically reduce the length of the hospital stay.

A previously healthy 6-month-old boy presents with upper body weakness. He cannot hold his eyes open, pupils do not react, and he cannot hold his head up. What is the proper treatment? (A) Send him home on amoxicillin and clindamycin (to stop the toxin production quickly) (B) Give him a dose of equine anti-botulinum immunoglobulin (C) Offer monitored supportive care with antibiotics and human anti-botulinum immunoglobulin (D) Offer monitored supportive care with human anti-botulinum immunoglobulin (E) Offer monitored supportive care with no antibiotics and no antitoxin

*The answer is D.* This question essentially asks which of the bacteria listed are found in soil and capable of surviving past the boiling point of water 100°C (212°F). Members of the Bacillus and Clostridium geniuses are commonly found in soil and are able to survive high temperatures desiccation and chemical agents by forming spores. Sporeforming bacteria can be killed by autoclaving.

A sample of contaminated moist soil is heated to 100°C for 15 minutes. Which of the following bacteria is most likely to be recovered from the soil sample following heat exposure? A. Streptococcus pyogenes B. Listeria monocytogenes C. Escherichia coil D. Bacillus anthracis E. Brucella melitensis

*The answer is D.* Staphylococcal food poisoning is caused by Staphylococcus aureus and not Staphylococcus epidermidis. It is a result of the ingestion of food contaminated with S. Aureus thatthe bacterium has grown on and produced a heat-stable enterotoxin. This is a toxemia and not an infection and that is why no antibiotic intervention is necessary, as well as why it occurs so quickly (2-3 hours) following ingestion of the contaminated food. It also is self-limiting and usually resolves itself in 24 hours. Bacillus cereus also produces a rapid food poisoning (3-4 hours after the ingestion of contaminated food), but the usual food is rice because the organism is a spore former and the spores survive the heating of the rice. Clostridium botulinum also produces a food poisoning but it may take 36 hours to manifest its elf and the end result is a flaccid paralysis and not vomiting and diarrhea. Escherichia coli is also a well-known cause of food poisoning but it takes at least 24 hours to manifest itself because the organism must grow in the gut and produce enough enterotoxin to cause the observed vomiting and diarrhea.

At a state dinner, the menu included steak or fried chicken, baked potato or homemade potato salad, green beans, and a "green" salad. The salad dressing was either Italian or Russian. Dessert included either chocolate cake or apple pie. The beverage was water, iced tea, or coffee After 3 hours, only the diners who had eaten the potato salad, became violently ill with vomiting, stomach cramps, and/or diarrhea. It became immediately obvious that the source of the food poisoning was the potato salad. When the potato salad was sent to a clinical microbiology lab which of the following bacteria was isolated in large numbers? a. Bacillus cereus b. Clostridium botulinum c. Escherichia coli d. Staphylococcus aureus e. Staphylococcus epidermidi

*The answer is E.* Diphtheria is an acute bacterial disease that is rare in the United States due to widespread administration of the Diphtheria-Pertussis-Tetanus (DPT) childhood vaccine. The DPT vaccine contains diphtheria toxoid, which stimulates production of neutralizing antibodies against the binding component (B subunit) of the diphtheria exotoxin. Antibody binding prevents the exotoxin from attaching to host cell membrane receptors, thus preventing disease (choice E). Corynebacterium diphtheriae colonizes the or oropharynx of non-immunized children and can cause pharyngitis. Patients may have a dark pseudomembrane in the posterior pharynx (composed of C. diphtheriae bacteria leukocytes, fibrin and necrotic mucosal epithelial cells) on physical exam. This pseudomembrane is characteristically tightly adherent and will cause bleeding if avulsed. The bacteria, however, are not the direct cause of the major clinical sequelae of diphtheria; instead, it is the C. diphtheriae exotoxin that exerts these effects. Exotoxin is released into the bloodstream and deactivates elongation factor-2 (EF-2), a protein that facilitates the movement of the forming peptide chain on the human ribosome. This effectively inhibits human protein synthesis. The clinical consequence is cardiac and neural toxicity resulting in cardiomyopathy and heart failure as well as neuropathy, paralysis, coma and death in 10% of patients.

An 8-year-old male who recently moved to the United States from Asia is brought to the emergency room with throat pain and difficulty breathing. Soon after being admitted to the hospital, he dies of severe heart failure. In the laboratory, bacterial isolates from this patient's tonsils are found to cause rapid death of 8 out of 10 exposed guinea pigs. Two of the 10 experimental animals survive. The surviving animals most likely possess which of the following antibodies? A. IgA against adhesion proteins B. Opsonizing IgM C. Complement-fixing IgM D. Immobilizing lgG E. lgG against circulating proteins

*The answer is E.* Listeria monocytogenesis a Gram positive rod that produces a very narrow zone of beta-hemolysis on sheep blood agar, similar to the pattern produced by colonies of β-hemolytic Streptococci. L. monocytogenes shows tumbling motility at 22°C but can be cultured at temperatures as low as 4°C. It is a facultative intracellular parasite and the only Gram positive bacteria to produce lipopolysaccharide (LPS) endotoxin. Listeria can cause serious disease (meningitis septicemia) in newborns, pregnant women, the elderly and immunocompromised patients, but rarely causes disease in immunocompetent individuals. Macrophages ingest L. monocytogenes, which survives phagocytosis and can go on to multiply in the cytoplasm. In healthy individuals, the cell-mediated immune response stimulates the production of cytokines (interferon gamma, tumor necrosis factor-beta and interleukin-12) that induce a cytotoxic T-cell response and macrophage activation and killing of intracellular Listeria. In patients with compromised cell-mediated immunity, however, the organism survives and disease results (Choice E).

Blood cultures from a 54-year-old male recently diagnosed with Hodgkin's disease reveal motile Gram positive rods that produce a very narrow zone of β-hemolysis on sheep blood agar. Which of the following processes is most important in eliminating these bacteria from the body? A. Terminal complement cascade B. Eosinophil action C. Immunoglobulin secretion D. Neutrophil oxidative burst E. Cell-mediated immunity

*The answer is A.* Contamination of wounds is the most common route of infection by this organism. However, the symptoms of gastroenteritis associated with some strains of Clostridium perfringens are usually caused by contamination of food.

Clostridium perfringens infections are commonly associated with: A. contamination of wounds. B. antibiotic treatment. C. consumption of water contaminated with sewage. D. immunosuppression. E. preexisting lung disease.

*The answer is D.* The question stem describes Corynebacterium diphtheriae, a non-motile unencapsulated, Gram positive rod. The organisms are often found in clumps resembling Chinese letters or joined in V- or Y-shaped chains on microscopic examination. Their cytoplasm contains metachromatic granules that stain with aniline dyes like methylene blue. C. diphtheriae produces a classic two subunit AB exotoxin. The B (binding) subunit binds specifically to the heparin binding epidermal growth factor receptor found on cardiac and neural cells, and is responsible for the toxin's affinity for heart and nervous tissue. The B subunit induces endocytosis of the toxin, and the subsequently released A (active) subunit inhibits host cell protein synthesis by catalyzing the ADP-ribosylation of protein elongation factor 2 (EF-2). (EF-2 is necessary for tRNA to insert new amino acids into the growing protein chain during translation.) By inhibiting cell protein synthesis (choice D), the toxin causes cell death.

Clustered Gram positive bacteria are isolated from the tonsillar exudates of a 6-year-old boy. On microscopic examination, the bacteria have polar granules that stain deeply with aniline dyes. The pathogenicity of these organisms is the result of: A. Widespread T-cell activation B. Blockade of neurotransmitter release C. Activation of electrolyte transport D. Impairment of protein synthesis E. Cellular membrane disruption

*The answer is C.* Intravascular catheters are indispensable in modern-day medical practice. Although these devices provide necessary vascular access, their use puts patients at risk for local and systemic infections. A total of 50000 to 20000 central venous catheter-associated bloodstream infections have been estimated to occur annually in the US. The most common pathogens isolated from hospital-acquired bloodstream infections are: coagulase-negative Staphylococci, S. aureus, Enterococcus, Escherichia co/i, Enterobacter, Pseudomonas aeruginosa, Klebsiella pneumoniae, and Candida.

Epidemiological analyses show a significant increase in staphylococcal infections (bacteremia and sepsis) in several large city hospitals over the last 20 years. This increase in the infection incidence most closely correlates with the increased use of: A. Prophylactic antibiotics B. Cyclosporine immunosuppression C. Intravascular devices D. Laparoscopic surgeries E. Blood transfusions

*The answer is D.* The question stem describes the inoculation of Gram-positive organisms into an experimental animal and treatment with an antibiotic that causes these organisms to be destroyed when exposed to a hypotonic solution. Recall that Gram positive organisms have a cytoplasmic membrane composed of a phospholipid bilayer as well as a peptidoglycan cell wall outside of that cell membrane. The peptidoglycan cell wall provides the shape of the bacterium as well as resistance to osmotic stress. Under normal circumstances, Gram-positive organisms would not be destroyed by variations in tonicity within a certain range due to their intact peptidoglycan cell wall. Because the organisms described in this question were destroyed by placement in a hypotonic solution one can infer that there was damage to the peptidoglycan cell wall by the antibiotic used. The only antibiotic listed that acts against cell wall synthesis is Cefuroxime (Choice D). Cefuroxime is a second generation cephalosporin. Cephalosporins are Beta lactam antibiotics and are related to penicillins. Vancomycin is another example of an antibiotic that inhibits cell wall synthesis.

Gram-positive bacteria are inoculated under the skin of experimental animals and then the infection is treated with antibiotics. Bacteria isolated from the injection site several days later assume a spherical configuration when placed in an isotonic solution and disintegrate rapidly when placed in a hypotonic solution. Which of the following antibiotic was most likely used in this experiment? A. Chloramphenicol B. Azithromycin C. Ciprofloxacin D. Cefuroxime E. Doxycycline

*The answer is B.* Seeing gram positive cocci in clusters on gram stain indicates the presence of a Staphylococcus species. Protein A is part of the outer peptidoglycan layer of coagulase positive Staphylococcus species (i.e. Staphylococcus aureus). Protein A binds non specifically with the Fc portion of lgG at the complement-binding site thus preventing the activation of complement. This results in decreased production of C3b leading to impaired opsonization.

Histopathology of an exudate from a patient with a localized skin infection demonstrates gram-positive cocci in clusters. The defensive protein bound to this organism's peptidoglycan cell wall does which of the following? A. Cleaves IgA B. Binds the Fc portion of IgG C. Interacts with MHC type II antigens D. Causes hemolysis E. Activates complement

*The answer is C.* Diphtheria is an acute toxin-mediated disease, but not all strains of 0. diphtheriae express the disease-causing exotoxin. C. diphtheriae acquires virulence via bacteriophage-mediated "infection" with the Tox gene (choice C), which codes for the diphtheria AB exotoxin. The bacteriophage responsible is called Corynephage beta. The phage Tox gene incorporates into the bacterial chromosome as a prophage and codes for toxin production by C. diphtheriae. This process, whereby a bacteriophage infects a host bacterium and integrates its genome into the host bacterium's genome, is termed lysogenization.

Non-pathogenic strains of Corynebacterium diphtheriae can acquire pathogenicity and thus the ability to cause severe pseudomembranous pharyngitis through which of the following mechanisms? A. Transformation permitting capsule formation B. Bacterial conjugation permitting pili expression C. Phage conversion permitting exotoxin production D. Chromosomal mutation permitting endotoxin production F. Environmental influences permitting toxin synthesis

*The answer is B.* Tetanus antitoxin is an essential reagent in wound prophylaxis and the treatment of clinical disease. It neutralizes only that toxin that has not bound the neuronal receptors. In the case of gas gangrene (A), numerous clinical studies have shown no advantage in the use of antitoxin preparations, and, presumably, the same would be true for necrotic enteritis and pseudomembranous colitis (C and D). Possible toxins among the gram-negative anaerobes are poorly described, and no therapeutic antisera are available (E).

Specific antitoxin is an important part of treatment in: A. gas gangrene. B. tetanus. C. necrotic enteritis. D. pseudomembranous colitis. E. Bacteroides and Prevotella infections.

*The answer is B.* Streptococcus are catalase-negative.The organisms are homofermentative i.e., the predominant end product of sugar fermentation is lactic acid.

Streptococcus are catalase-positive. a) True b) False

*The answer is A.* Currently, the most common form of botulism in the United States occurs in infants. Malignant food poisoning (C) was the first described form of infection and is probably the best known. A wound focus is rare (B). Clostridium botulinum is noninvasive and causes neither septicemia (D) nor cellulitis (E).

The most common form of infection caused by Clostridium botulinum in this country is: A. infant botulism. B. wound infection. C. food poisoning. D. primary septicemia. E. anaerobic cellulitis.

*The answer is A.* P. aeruginosa produces several extracellular products including exotoxin A, collagenase, elastase, fibrinolysin, phospholipase C and DNAse. These substances assist in its invasion and dissemination in human tissues. Diphtheria toxin acts in a similar manner to exotoxin, though they are structurally different. Both diphtheria toxin and exotoxin A ribosylates and inactivate elongation factor-2 (EF-2), halting human cell protein synthesis and causing cell death. Exotoxin A is a major virulence factor and responsible for the high mortality associated with P. aeruginosa septicemia.

Three cases of severe pharyngitis were reported in a community of immigrants. The patients had thick pharyngeal exudates, neck swelling and difficulty swallowing. One of them died from severe heart failure. The toxin responsible has a mechanism similar to the toxin of which of the following bacteria? A. Pseudomonas aeruginosa B. Staphylococcus aureus C. Clostridium difficile D. Clostridium botulinum E. Bordetella pertussis F. Vibrio cholerae

*The answer is B.* Gram stain and culture of this skin wound shows gram-positive cocci in clusters that do not respond to nafcillin but show sensitivity to vancomycin. This is a classic methicillin-resistant Staphylococcus aureus infection. S. aureus is one of the most common causes of skin infections in the United States. Most of these skin infections are minor, but S. aureus also can cause serious infections such as surgical wound infections, bloodstream infections in the presence of central intravenous catheters and pneumonia. Approximately 95% of S. aureus isolates in the United States produce β-lactamase (penicillinase), which causes resistance to penicillin; however, many S. aureus strains, while resistant to penicillin, remain susceptible to penicillinase-stable penicillins such as oxacillin, nafcillin and methicillin. Strains that are oxacillin, nafcillin and methicillin resistant historically termed methicillin-resistant S. aureus (MRSA)I are resistant to all β-lactam agents, including cephalosporins and carbapenems. Nafcillin (methicillin) resistance is mediated by alterations in the penicillin-binding protein (PBP) structure. PBPs are the enzymes involved in cell wall synthesis. Altered PBPs, particularly PB2a, have greatly reduced affinity for all β-lactam antimicrobial agents.

A 12-year-old male is treated with nafcillin for a skin infection and shows little response after two days of therapy. Pus microscopy shows Gram-positive cocci in clusters, and sensitivity testing demonstrates little response to nafcillin but good response to vancomycin. Which of the following would most likely explain treatment failure in this patient? A. Enzymatic degradation of the drug B. Poor interaction with binding proteins C. Mutation in RNA polymerase D. Active drug transport out of the cell E. Mutation in DNA gyrase

*The answer is D.* The minor skin infection mentioned in this question stem is impetigo, a skin infection caused by Streptococcus pyogenes (Group A Strep). S. pyogenesis a catalase negative beta-hemolytic, Gram-positive coccus that forms chains on microscopy. It is unique among beta-hemolytic streptococci because it is sensitive to bacitracin. Rheumatic fever and acute post-streptococcal glomerulonephritis (APSGN) are the late sequelae that can follow an acute group A streptococcal infection. The patient in this question stem is experiencing symptoms consistent with post-streptococcal glomerulonephritis.

A 17-year-old female presents to your office with dark urine and facial puffiness. She had a minor skin infection several weeks ago that resolved spontaneously. Microscopic examination of the urine sediment shows red blood cell casts. The organism responsible for this patient's symptoms would most likely demonstrate: A. Catalase positivity B. Growth in 6.5% NaCl C. Bile solubility D. Bacitracin sensitivity E. Optochin sensitivity

*The answer is D.* Fever, vomiting diarrhea, muscle pain and erythroderma are the symptoms of Toxic Shock Syndrome (TSS). It can rapidly progress to severe hypotension and multisystem dysfunction. Desquamation particularly on the palms and soles can occur 1-2 weeks after the onset of illness. TSS has been associated with the use of tampons, nasal packing etc. Staphylococcus aureus strains producing toxic shock syndrome toxin (TSST-1) are responsible for most cases of TSS. TSST acts as a superantigen. It is called a superantigen because in contrast to usual antigen which activates few helper T cells it activates large numbers of helper T cells. These toxins interact with major histocompatibility complex molecules on antigen presenting cells and the variable region of the T lymphocyte receptor to cause a nonspecific widespread activation of T lymphocytes. Activation of T cells is responsible for the release of interleukin-2 (IL-2) from the T cells and IL-1 and TNF from macrophages. These interleukins cause capillary leakage, circulatory collapse hypotension shock fever, skin findings, and multiorgan failure. *Educational Objective:* Enterotoxins Exfoliative Toxins and Toxic Shock Syndrome Toxin (TSST-I) are the toxins with superantigen activity. Superantigens interact with major histocompatibility complex molecules on antigen presenting cells and the variable region of the T lymphocyte receptor to cause nonspecific "widespread" activation of I-cells resulting in the release of interleukin-2 (IL-2) from the T cells and IL-i and TNF from macrophages. The immune cascade in turn is responsible for the effects of TSS.

A 23-year-old Caucasian female is brought to the ER with fever, vomiting, diarrhea and muscle pain. Her blood pressure is 90/50 mm Hg and pulse is 120/mm. Physical examination reveals erythroderma, and pelvic exam reveals a tampon in the vagina. The activation of which of the following cells is primarily responsible for this patient's condition? A. Mast cells and eosinophils B. Basophils and macrophages C. Neutrophils and B lymphocytes D. Macrophages and T lymphocytes E. Platelets and mast cells

*The answer is A.* Many patients with sickle cell disease come to the ER frequently with painful, vasoocclusive crises; these crises usually respond well to oxygen, IV fluids, and high-dose narcotics. The above presentation is different from a run-of-the-mill crisis because the patient is febrile, which indicates infection. The vaso-occlusion that is associated with sickle cell anemia not only causes the painful "sickle cell crises," it also causes a relative immune deficiency because the spleen suffers widespread infarction. This functional asplenia puts sickle cell patients at an increased risk of infection by encapsulated organisms such as Neisseria, Haemophilus, Streptococcus pneumoniae, and Salmonella species. Certain vaccinations are routinely given to patients with sickle cell disease and patients who are asplenic for other reasons (i.e. trauma). These vaccinations are: the Pneumovax for S. pneumoniae, Hib for H. influenzae type b, and the Meningitis polysaccharide capsular vaccine for N. meningitidis. Salmonella possesses a special capsule called the "Vi antigen" (Vi stands for virulence) which protects the bacterium from opsonization and phagocytosis. Salmonella is the most common cause of osteomyelitis in patients with sickle cell anemia (choice A), followed by E. coli (K antigen capsule), then S. aureus. An increased risk of osteomyelitis is present because vaso-occlusive crises cause focal areas of bone necrosis; within which bacteria can easily establish infection.

A 24-year-old African-American female presents to the emergency room with fever, malaise, and intense pain over her right thigh. She has a long history of frequent presentation to the hospital with painful crises. Usually she is admitted and treated with supplemental oxygen and intravenous narcotics. This episode of pain is very different because she is febrile (103.0F) with exquisite tenderness over the right thigh, as well as local warmth. You send her for an MRI, and based on those results you wish to begin antibiotics promptly. Which of the following organisms is most likely causing her clinical presentation? A. Salmonella B. Escherichia C. Klebsiella D. Staphylococcus E. Shigella

*The answer is E.* The patient shows signs of toxic shock syndrome. Toxic shock syndrome as defined in the outbreak of the late '70s and early '80s included an erythematous/peeling rash (not purpuric) and was caused by overproduction of toxic schock syndrome toxin (TSST)-1 by colonizing S. aureus triggered by something in hyperabsorbent tampons. Many signs and symptoms are the results of the superantigen activity of TSST, which activates a whole subclass of T cells, causing overproduction of cytokines. S. saprophyticus is a frequent cause of cystitis in women, but is not associated with toxic shock syndrome.

A 32-year-old woman became ill 4 days after the onset of her menstrual period. She presented in the emergency room with fever (104°F; normal = 98.6°F), elevated white blood cell count (16,000/mm3; normal = 4,000 to 10,000/mm3), and an erythematous, sunburn-like rash on her trunk and extremities. She complained of fatigue, vomiting, and diarrhea. She had recently eaten at a fast-food restaurant, but otherwise had prepared all her meals at home. The patient described !most likely has: A. staphylococcal food poisoning. B. scalded skin syndrome. C. infection with a Staphylococcus saprophyticus. D. chickenpox. E. toxic shock syndrome.

*The answer is C.* The Staphylococci are Gram-positive cocci that form clusters, pairs and, rarely, short chains. The catalase test (with 3% hydrogen peroxide) differentiates Streptococci (catalase-negative) from Staphylococci (catalase-positive). The ability to clot blood plasma (slide and tube coagulase tests) separates Staphylococci into two groups: the coagulase positive Staphylococci which constitutes the most pathogenic species Staphylococcus aureus, and coagulase negative staphylococci (CNS) which constitutes S. epidermidis, S. Saprophyticus, S. haemolyticus, and 30+ other species. The coagulase-negative staphylococci exist as part of the normal flora on the skin and in the throat and nose, and only some species can cause infections. S. saprophyticus a common cause of urinary tract infection. It is responsible for almost half of all UTIs in sexually active young women. S. saprophyticus is resistant to novobiocin. When catalase-positive, coagulase-negative gram- positive cocci in clusters are isolated from urine specimens of the above group of patients the laboratory performs a novobiocin test to distinguish this organism from other similar pathogens.

A 24-year-old female presents to your office with burning urination, urgency and frequency. She is sexually active. Urine cultures show catalase-positive, gram-positive cocci. The organism responsible for this patient's symptoms is most likely to be: A. Coagulase positive B. Hemolytic C. Novobiocin resistant D. DNase positive E. Yellow pigment producer

*The answer is A.* Staphylococcus aureus is capable of producing a highly heat-stable protein toxin called enterotoxin that causes staphylo enterotoxemia, a syndrome characterized by nausea, vomiting and abdominal cramps following ingestion of preformed exotoxin. S. aureus contaminates food by direct inoculation from food handlers who are frequently asymptomatic carriers. Subsequently the food is allowed to sit at room temperature for an extended period of time (i. e. at a picnic / pot luck), and S. aureus is able to multiply and produce exotoxin. This preformed exotoxin is then ingested by the patient resulting in symptoms. Because this is a preformed exotoxin, there is no person-to-person transmission, but outbreaks can occur with many people eating the same contaminated food. It is also the reason for the rapid onset of symptoms (usually less than 6 hours). Poultry and egg products, meat and meat products, egg, tuna, chicken, potato and macaroni salads, cream-filled pastries, and milk and dairy products are the foods that are frequently incriminated in staphylococcal food poisoning. The most frequently tested food item is a mayonnaise-containing food like potato or macaroni salad. A similar clinical picture of rapid-onset nausea, vomiting, abdominal cramping and retching can be seen in food poisoning caused by Bacillus cereus which also contaminates food with a preformed heat-stable exotoxin. B. cereus characteristically econtaminates starchy foods such as rice (reheated fried rice).

A 24-year-old male presents to your ER with severe nausea and vomiting two hours after consuming a mayonnaise- containing salad at an outdoor picnic. His condition improved significantly in several hours. You inform the patient that his symptoms were caused by which of the following? A. Exotoxin formed prior to ingestion B. Exotoxin formed after ingestion C. Endotoxin D. Bacterial invasion of gut mucosa E. Parasitic Infection

*The answer is B.* The woman is most likely experiencing listerosis. Listeriae are common in the gastrointestinal tract and milk of cattle, but are normally killed by pasteurization. Unpasteurized milk was presumably consumed at the farm.

A 26-year-old woman, 8 months pregnant, visits her obstetrician complaining of fever, myalagia and backache of recent onset. Three weeks earlier, the patient had been a weekend guest at a rural farmhouse, where all the food was reported to be "unprocessed" and "natural." A culture of the patient's blood shows gram-positive rods that are catalase positive and display a distinctive tumbling motility in liquid medium. What is the most likely source of the woman's infection? A. Well-done roast beef B. Fresh, raw cow's milk C. Home-baked bread D. Homemade applesauce E. Baked apple pie

*The answer is C.* H. influenzae along with Streptococcus pneumoniae are the most frequent causes of acute otitis media. This infection is primarily seen in children between 6 months and 12 years of age. More than 90% of H. influenzae strains isolated from middle ear aspirates of infected children are nontypable; the remaining 10% are H. influenzae type b. H. influenzae is classified or typed, based on the antigens in its polysaccharide capsule, and strains of H. influenzae that do not produce a capsule are referred to as nontypable. Nontypable H. influenzae strains are part of the upper respiratory tract normal flora, but they can also cause otitis media, sinusitis and bronchitis in adolescents and adults as well as children and vaccination with the H. influenzae type b (Hib) vaccine does not confer immunity to any strain except type b. It is recommended that all children receive the Hib vaccine beginning at 2 months of age as H. influenzae type b.

A 3-year-old male experiences several episodes of otitis media. Middle ear exudate cultures grow H. influenzae although the patient's immunizations are up-to-date. Which of the following explains H. influenzae infection in this patient despite immunizations? A. No vaccine is effective against H. influenzae B. The strains responsible for this patient's disease produce exotoxin C. The strains responsible for this patient's disease do not produce capsule D. H. influenzae infection despite immunizations means defective cellular immunity E. H. influenzae infection despite immunizations signifies defective neutrophil function

*The answer is B.* Tetanospasmin is a potent neurotoxin produced by the vegetative cells of Clostridium tetani. C. tetani spores are commonly found in soil and may contaminate injured tissue, classically puncture wounds from soil-contaminated objects. The low oxygen tension and high availability of nutrients in a puncture wound provides a localized anaerobic environment for the germination of spores growth of vegetative cells and production of toxin within bacterial cells. Upon bacterial cell autolysis, the toxin is released and gains access to the motor neuron at the axon terminal. The toxin then travels up the motor neuron axon by retrograde transport to the spinal cord and medulla. The organism itself does not travel, but rather remains at the local wound site. Tetanus toxin inhibits the inhibitory interneurons that normally regulate the motor neuron, leading to unregulated action of the motor neuron and tetany. Classic tetanus findings include jaw stiffness due to spasmodic contraction of the masseter muscle (trismus) and sustained contraction of the facial muscles producing a bizarre "smiling" appearance (risus sardonicus). Spasmodic contraction further extends to the back and neck leading to opisthotonos, and respiratory muscle involvement can lead to respiratory failure.

A 32-year-old migrant ranch worker is brought to the emergency room with painful paroxysmal involuntary muscle contractions involving mainly the jaw, neck and trunk. The causative substance has most likely traveled by which of the following routes? A. Wound — neuron axons — salivary glands B. Wound — motor neuron axons — spinal cord C. Fibrinous exudate —a systemic circulation — cortical neurons D. Food — systemic circulation — meninges E. Food —a systemic circulation —a peripheral nerves

*The answer is C.* This patient likely experienced splenic rupture secondary to abdominal trauma two years ago with splenic remnants subsequently removed during the laparotomy. The spleen is a part of the systemic lymphoid system that receives roughly 6% of the cardiac output. Its red pulp filters and destroys aged erythrocytes and serves as an emergency source of erythrocytes, granulocytes and platelets that can be instantly delivered into the circulation when needed. The white pulp consists of lymphoid tissue with T- and B-cell regions. Splenic mononuclear phagocytes and dendritic cells ingest circulating bacteria particularly un opsonized organisms, and present them to B- and T-cells residing in the spleen in order to quickly mount an immune response. Nearly half of the body's total immunoglobulins are produced by the splenic B lymphocytes. Asplenic patients are predisposed to sepsis with encapsulated bacteria such as S. pneumoniae, H. influenzae, and N. meningitidis due to a decreased ability to recognize and clear these organisms. Vaccination against these three organisms is recommended for asplenic patients.

A 34-year-old male dies from a fulminant S. pneumoniae infection. His past medical history was significant only for a motor vehicle accident two years ago in which he sustained blunt abdominal trauma and required emergent laparotomy. Impairment of which of the following protective mechanisms most likely contributed to the severity of this patient's infection? A. Complement fixation B. Interferon release C. Bacterial clearance D. Opsonization E. Intracellular killing

*The answer is D.* Botulism is a rare but serious paracytic illness caused by a nerve toxin produced by the bacterium Clostridium botulinum. There are three main types of botulism. Foodborne botulism results when an individual eats foods containing the botulism toxin. Wound botulism is the result of toxin production in a wound infected with C. botulinum. Infant botulism occurs when a baby consumes C. botulinum spores which then mature into vegetative toxin-producing cells in the intestine. (Contaminated honey is a frequent cause of infant botulism.) All forms of botulism can be fatal and are considered medical emergencies. Botulinum neurotoxin is among the most toxic substances known. The anaerobic environment within a can of food contaminated by C. botulinum spores allows spore germination and organism growth. The botulinum toxin produced by the vegetative C. botulinum bacteria remains intracellular until autolysis releases the potent neurotoxin into the food. (This toxin IS NOT actively secreted by the bacteria.) The toxin is readily destroyed by heating but if food containing the pre-formed neurotoxin is not cooked well the toxin will be able to exert its effects. The most common clinical manifestations are diplopia, dysphagia and dysphonia (three 'Ds'), which occur within 12-46 hours of neurotoxin consumption.

A 34-year-old male is hospitalized with difficulty swallowing and blurred vision of sudden onset. He has never had symptoms like this before. Routine nerve stimulation studies show normal nerve conduction velocity but decreased compound muscle action potential. The patient should be carefully questioned about: A. Missed vaccinations B. Recent travel C. Unprotected sexual contacts D. Home-canned food consumption E. Family history F. Recent antibiotic use

*The answer is B.* About 95% of cases of gas gangrene are due to Clostridium perfringens. The spores of this Gram positive bacillus are abundant in soil, and patients suffering penetrating injuries can be inoculated with spores. Upon entering the host, the C. perfringens spores germinate in the anaerobic environment into vegetative toxin-producing cells. The toxin is a phospholipase that attacks cell membranes and causes cell death. Extensive tissue damage, necrosis, and reduction of blood supply to the affected area result, and the disease continues to spread in the enlarging anaerobic environment. The organism rapidly metabolizes carbohydrate, producing a significant amount of gas that can be visualized on plain film radiographs. Treatment involves emergent debridement and intravenous antibiotics, but even with prompt therapy the prognosis of clostridial myonecrosis, or gas gangrene, is poor and tissue loss is often considerable. C. perfringens can also cause a late-onset food poisoning characterized by a transient watery diarrhea. This gastroenteritis is caused by toxin formed when large quantities of clostridial spores are ingested. The spores germinate in the digestive tract and then begin to elaborate toxin, hence the diseases delayed onset. (This is in contrast to the early-onset food poisoning caused by the preformed toxins of S. aureus and B. cereus.)

A 35-year-old male hospitalized with extensive trauma following a tractor accident complains of severe pain in his injured right leg. The leg swells rapidly, and radiographs reveal gas in the tissues. The microorganism most likely responsible for this patient's condition is also commonly associated with: A. Persistent cough B. Transient watery diarrhea C. Polyarthritis D. Urinary frequency and burning E. Relentless headaches and nausea

*The answer is B.* The most probable etiology of bacterial endocarditis involving the tricuspid valve is illicit intravenous drug use, which can introduce skin organisms into the venous system that then attack the tricuspid valve. Staphylococcus aureus accounts for between 60% and 90% of cases of endocarditis in intravenous drug users. The endocarditis associated with congenital heart disease (choice A) typically involves either damaged valves or atrial or ventricular septal defects. The tricuspid valve is not particularly vulnerable. Rheumatic fever (choice C) most commonly damages the mitral and aortic valves, and tricuspid damage is usually less severe and seen only when the mitral and aortic valves are heavily involved. Consequently, secondary bacterial endocarditis involving only the tricuspid valve in a patient with a history of rheumatic fever would be unusual. Rheumatoid arthritis (choice D) is not associated with bacterial endocarditis. Systemic lupus erythematosus (choice E) can produce small, aseptic vegetations on valves, but is not associated with bacterial endocarditis.

A 37-year-old female presents to the emergency room with a fever. Chest x-ray shows multiple patchy infiltrates in both lungs. Echocardiography and blood cultures suggest a diagnosis of acute bacterial endocarditis limited to the tricuspid valve. Which of the following is the most probable etiology? A. Congenital heart disease B. Illicit drug use C. Rheumatic fever D. Rheumatoid arthritis E. Systemic lupus erythematosus

*The answer is C.* All the organisms listed are Gram-positive cocci. Staphylococci are distinguished from Streptococci and Enterococci by catalase production. Thus, the organism is a species of Staphylococcus. Coagulase production distinguishes S. aureus from the other staphylococcal species, which are often referred to as coagulase-negative staphylococci. Thus, the etiological agent in this case is S. epidermidis.

A 48-year-old man has been hospitalized in intensive care for 3 weeks following an automobile accident. He develops a fever due to colonization of an intravenous line with Gram-positive cocci that are catalase-positive and coagulase-negative. What is the most likely cause of this man's fever? (A) Enterococcus faecalis (B) Staphylococcus aureus (C) Staphylococcus epidermidis (D) Streptococcus mutans (E) Streptococcus pyogenes

*The answer is B.* The above slide illustrates wedge-shaped hemorrhagic pulmonary infarcts. Pulmonary infarcts are almost always hemorrhagic due to the dual blood supply to the lungs (pulmonary and bronchial arteries). The question stem describes a young male patient with a history of intravenous drug abuse dying of sepsis. Intravenous drug abusers most commonly form endocarditis of the right heart. Recall that S. aureus is the most common endocarditis associated with IVDU. Pulmonary infarction in this case likely resulted from septic embolization from the tricuspid valve to a distal pulmonary artery. *Educational Objective:* S. aureus is the most common cause of tricuspid endocarditis in intravenous drug users. P. aeruginosa is the second most common cause in this patient population. These patients can develop multiple septic emboli in lungs. Pulmonary infarcts are almost always hemorrhagic due to the dual blood supply to the lungs (pulmonary and bronchial arteries).

A 38-year-old male intravenous drug user hospitalized for high-grade fever, fatigue and dyspnea dies in the intensive care unit (ICU). His lung autopsy findings are shown on the slide below. This patient most likely suffered from: A. Mycotic aortic aneurysm B. Tricuspid valve endocarditis C. Severe small airway obstruction D. Miliary tuberculosis E. Venous thromboembolism

*The answer is A.* The baby has infant botulism (floppy baby syndrome), which is due to germination of Clostridium botulinum spores (found in honey) in the baby's gastrointestinal tract. Patients improve when honey is removed from the diet. This disorder is most common in children under the age of six months; older children and adults do not appear to be vulnerable to this form of botulism, but are susceptible to botulism caused by ingestion of preformed toxin. Clostridium difficile (choice B) causes pseudomembranous colitis, especially after antibiotic therapy. Clostridium perfringens (choice C) causes gas gangrene and gastroenteritis, and it is not associated with ingestion of honey. Clostridium tetani (choice D) causes tetanus, and does not cause a food-borne illness in infants. Corynebacterium diphtheriae (choice E) causes diphtheria in susceptible individuals.

A 4-month-old infant presents with failure to thrive, progressive muscular weakness, and poor head control. On questioning, the mother states that she typically feeds the baby soy-based formula sweetened with honey. Which of the following organisms is most likely to be responsible for the child's presentation? A. Clostridium botulinum B. Clostridium difficile C. Clostridium perfringens D. Clostridium tetani E. Corynebacterium diphtheriae

*The answer is B.* A patient presenting with headache, fever and nuchal rigidity should immediately raise suspicion for meningitis. In bacterial meningitis CSF analysis typically shows elevated neutrophils, decreased glucose, and elevated protein. The morphology of any observed bacteria on CS Gram stain provides an excellent preliminary identification of the pathogen (whereas CSF cultures take time to grow). Streptococcus pneumoniae is the most common cause of bacterial meningitis in adults. On CSF Gram stain, one would see lancet-shaped Gram positive cocci in pairs. S. pneumoniae meningitis often follows a pulmonary infection or mild upper respiratory infection. Alcoholics, sickle cell anemia patients, asplenic individuals, or those in generally poor health are at risk for meningitis caused by S. pneumoniae.

A 44-year-old male presents to the emergency room complaining of severe headache, nausea, and fever. Lumbar puncture and cerebrospinal fluid (CSF) examination reveal the following: Opening pressure 300 cm H20 Glucose 20 mg/dL Protein 200 mg/dL RBC 4/mm3 WBC 360/mm3 Neutrophils 90% Lymphocytes 10% CSF gram stain would most likely reveal: A. Bean-shaped Gram negative cocci in pairs B. Lancet-shaped Gram positive cocci in pairs C. Gram positive cocci in clusters D. Non-motile Gram negative coccobacilli E. Motile Gram positive rods

*The answer is E.* This cattle rancher is suffering from cutaneous anthrax, which is an occupational hazard. The scab-like wound is called an eschar and results from localized edema and tissue destruction caused by the two toxins produced by Bacillus anthracis. The microbiological characteristics of the organism are consistent with a diagnosis of B. anthracis infection. The other microorganisms do not have the chacteristics described.

A 45-year-old cattle rancher presents to his physician with a wound on his forearm that resembles a large scab. Samples collected from the wound were cultured and examined. The bacteria recovered were Gram positive, nonmotile rods with square ends. The cultured bacteria formed irregularly shaped, nonhemolytic colonies on blood agar plates and individual cells from the plates had a centrally located spore. What is the most likely cause of this infection? A. Listeria monocytogenes B. Staphylococcus aureus C. Legionella pneumophila D. Corynebacterium diphtheriae E. Bacillus anthracis

*The answer is A.* The occupational history of the patient wool processing, is an important risk factor to elicit on history to aid in making the diagnosis of pulmonary anthrax. Pulmonary anthrax is also known as "woolsorters disease" because exposure from handling animal products such as animal hair, infected animal hides, hide-based clothing products or wool has been associated with infection by Bacillus anthracis. Goat hair is the most commonly implicated exposure in contraction of anthrax. The spores of B. anthracis are very small, once they are inhaled they enter the alveoli and are ingested by macrophages. From the lung the organisms rapidly move to mediastinal lymph nodes and cause hemorrhagic mediastinitis. Once the spores germinate into vegetative cells they will begin to produce the three-part anthrax toxin and clinical symptoms will rapidly follow. Symptoms initially only consist of myalgia fever and malaise but rapidly progress to hemorrhagic mediastinitis (widened mediastinum on chest x-ray), bloody pleural effusions, septic shock and death. Bacillus anthracis produces an antiphagocytic capsule that is required for pathogenicity. The capsule is unique in that it contains D-glutamate instead of polysaccharide. B. anthracis is an aerobic organism that can be grown on standard culture media and forms non hemolyzing adherent colonies. On microscopy it forms long chains that are described as being "serpentine" or "medusa head" on appearance.

A 46-year-old female presents to the emergency room with fever, chest pain and hemoptysis. She came to visit her family from Pakistan where she works in a goat wool processing center. Chest x-ray shows widened mediastinum. Sputum and blood cultures reveal large gram positive rods that form 'medusa head colonies on standard media. The bacteria isolated from this patient most likely produce: A. Antiphagocytic D-glutamate capsule B. Antiphagocytic polysaccharide capsule C. lgG-binding outer membrane protein D. Intracellular polyphosphate granules E. Peritrichous flagella

*The answer is A.* Staphylo coccus epidermidis produces a polysaccharide, referred to as a slime layer or biofilm, that surrounds the growing colony of organisms. The polysaccharide enables the bacteria to attach firmly to medical devises such as catheters as well as prosthetic joints or heart valves. The biofilm also protects the organisms from opsonins and phagocytes and slows the penetration of antibiotics. Panton-Valentine leukocidin is a cytotoxin found in some strains of Staphylococcus aureus, particularly community-acquired MRSA. Pyrogenic exotoxin and streptokinase are important virulence factors of group A streptococci. Some strains of enterococci are noted for vancomycin resistance.

A 48-year-old man has been hospitalized in intensive care for 3 weeks following an automobile accident. He develops a fever due to colonization of an intravenous line with Gram-positive cocci that are catalase-positive and coagulase-negative. What is the major virulence factor of the organism in the above case? (A) Biofilm production (B) Panton-Valentine leukocidin (C) Pyrogenic exotoxin (D) Streptokinase (E) Vancomycin resistance

*The answer is C.* Catalase positivity separates staphylococci from streptococci. Coagulase positivity separates S. aureus from the socalled "coagulase-negative staph" including S. epidermidis and S. saprophyticus. S. aureus is b-hemolytic. Bacitracin sensitivity helps distinguish group A streptococci from other streptococcal species. Lactose fermentation is used to distinguish members of the Enterobacteriaceae group. Urea hydrolysis is a notable characteristic of Proteus sp. and Helicobacter pylori.

A 5-day postoperative patient develops a high fever. An IV catheter is removed and culture of the tip reveals Gram-positive cocci believed to be Staphylococcus aureus. Which of the following laboratory test results would further support this belief? (A) α-Hemolysis on blood agar (B) Bacitracin sensitivity (C) Coagulase positivity (D) Lactose fermentation (E) Urea hydrolysis

*The answer is A.* This 5-month-old baby is consuming honey, a food notorious for contamination with C. botulinum spores. In studies, more than 12% of tested honey samples contained C. botulinum spores. Infant botulism results when a baby consumes C. botulinum spores, which then germinate in the infant Gl tract. Intracellular toxin production bacteriolysis resulting in toxin release, and mild systemic absorption of toxin ensue. In infant botulism, constipation usually precedes the characteristic signs of neuromuscular paralysis by a few days or weeks. Other symptoms include mild weakness, lethargy, and reduced feeding. Some infants, however, show more severe symptoms such as weakened suck, swallowing, and crying: generalized muscle weakness: and diminished gag reflex. In severe cases, the generalized muscle weakness and loss of head control can cause the infant to appear 'floppy." (In contrast, adult botulism, which results from ingestion of preformed toxin, is almost always very severe.) Infant botulism can usually be diagnosed based on the patient's clinical presentation and food consumption history. Culture and isolation of the organism and bioassay of toxins are time-consuming procedures, but rapid in vitro procedures have been developed for the detection of types A, B, E, and F botulinum toxin-producing organisms and their toxins. The tests are based on ELISA methodology and polymerase chain reaction techniques.

A 5-month-old Hispanic boy is brought to the ER with complaints of poor feeding weakness and complete loss of extremity muscle tone. All of his vaccinations are up to date and there is no significant past medical history. He receives formula as his sole source of nutrition with the exception of occasional fruit juice and honey. He has also received vitamin D supplementation. Which of the following tests is most likely to establish the diagnosis in this patient? A. Stool for bacterial toxins B. Blood for liver enzymes C. Blood for viral titers D. Urine for glucose and ketones E. Urine for amino acids

*The answer is A.* This child has diphtheria, a disease caused by the C. diphtheriae AB-exotoxin. The A (active) subunit of this exotoxin transfers a ribose residue from NAD to a histidine on elongation factor-2 (EF-2). The EF-2 protein is essential for peptide chain translocation on the ribosome in the process of translation. Ribosylation inactivates EF-2, thus inhibiting protein synthesis and causing cell death.

A 5-year-old child who recently moved from Eastern Europe is brought to the emergency room with difficulty breathing and a low-grade fever. Physical examination reveals a grayish pharyngeal exudate, enlarged cervical lymph nodes and partial soft palate paralysis. Bacteria isolated from the pharyngeal exudate demonstrate exotoxin production in the laboratory. This exotoxin acts via the following mechanism: A. Intracellular protein ribosylation B. Degradation of lecithin C. Increased sensitivity to histamine D. Inactivation of the 60s ribosomal subunit E. Autoimmune activation by antigenic mimicry

*The answer is C.* Myocarditis can occur about 1-2 weeks following the onset of diphtheria. The diphtheria toxin interacts with receptors on myocardial cells resulting in the inhibition of protein synthesis. The heart is particularly susceptible to the effects of the toxin as myocytes express high levels of the toxin receptor and the toxin's effect is manifested as severe impairment of contractility. Myocarditis is seen in up to 60% of the patients. Death due to diphtheria is usually because of blockage of air passages by the pseudomembrane or myocarditis.

A 5-year-old girl from an orphanage in Ukraine was about to leave for the United States with her adoptive parents when she developed a fever, sore throat, cough, and malaise. A few days later, she was brought to the emergency room in respiratory distress. Physical examination revealed pronounced, bilateral cervical lymphadenopathy, inflamed pharyngeal area, and a thick, grayish, leathery exudate on the tonsils as shown in the photograph. Part of the exudate had become loose revealing bleeding of the underlying mucosal tissue. The etiologic agent was isolated on tellurite medium. Ten days after the onset of illness, the patient in the above case deteriorated and she subsequently died from a complication of her infection. What was the most likely cause of her death? (A) Encephalitis (B) Kidney failure (C) Myocarditis (D) Pneumonia (E) Sepsis

*The answer is E.* The case is descriptive of diphtheria caused by Corynebacterium diphtheriae. This bacterium produces a toxin that inhibits protein synthesis by inactivating elongation factor-2. It does not express any of the other virulence factors listed. It is not known to secrete an IgA protease as does Neisseria. As a Gram-positive organism, C. diphtheriae lacks lipopolysaccharide. Streptococcus pyogenes produces M protein as an important virulence factor. C. diphtheriae is not known to produce coagulase as does Staphylococcus aureus.

A 5-year-old girl from an orphanage in Ukraine was about to leave for the United States with her adoptive parents when she developed a fever, sore throat, cough, and malaise. A few days later, she was brought to the emergency room in respiratory distress. Physical examination revealed pronounced, bilateral cervical lymphadenopathy, inflamed pharyngeal area, and a thick, grayish, leathery exudate on the tonsils as shown in the photograph. Part of the exudate had become loose revealing bleeding of the underlying mucosal tissue. The etiologic agent was isolated on tellurite medium. What virulence factor accounts for the pathogenesis of this infection? (A) Immune evasion following production of IgA protease (B) Lipopolysaccharide-induced inflammation (C) M protein-mediated resistance to phagocytosis (D) Secretion of coagulase to facilitate invasion into mucosa (E) Toxin-mediated inhibition of protein synthesis

*The answer is B.* The scenario emphasizes the differentiating characteristics of staphylococci. The Staphylococci are Gram-positive cocci that most commonly form clusters, but can also form pairs and rarely short chains. The catalase test (with 3% hydrogen peroxide) differentiates Streptococci (catalase-negative) from Staphylococci (catalase-positive). The ability to clot blood plasma (slide and tube coagulase tests) separates Staphylococci into two groups: the coagulase positive Staphylococci species, which consists of the most pathogenic species Staphylococcus aureus, and coagulase-negative staphylococci (CNS) which includes S. epidermidis, S. haemolyticus, S. saprophyticus, and 30+ other species. Among coagulase-negative staphylococci, S. epidermidis is a ubiquitous skin contaminant. S. epidermidis is capable of causing an opportunistic infection associated with foreign bodies: it is the most common cause of endocarditis in patients with prosthetic valves and septic arthritis in patients with prosthetic joints.

A 52-year-old male is having low-grade fevers after recent replacement of his aortic valve. Repeated blood cultures grow gram-positive cocci. Which of the following characteristics is most consistent with Staphylococcus epidermidis as the cause of this patient's symptoms? A. Catalase negative B. Coagulase negative C. Mannitol fermenting D. Novobiocin resistance E. Yellow pigment production

*The answer is B.* Humans become infected with Bacillus anthracis most commonly by exposure through contact with infected animals or animal products or through use of B. anthracis as a biological weapon. For this reason an occupational history of exposure to animals or animal products is extremely important; if cutaneous anthrax is suspected in a patient without the risk of occupational exposure, then the potential for bioterrorism should be suspected and public health authorities contacted. The disease is rare in the Unites States; it is most common in areas where vaccination of livestock and animal carriers is not done or not possible. Growth of the vegetative organisms at the site of inoculation results in formation of a characteristic edematous wound with central necrosis leading to formation of a black eschar. B. anthracis spreads via lymphatics to the bloodstream, and the organism multiplies in the blood and tissue. Cutaneous anthrax is the most common form of this disease; pulmonary anthrax accounts for approximately 5% of cases, and gastrointestinal anthrax is a rare occurrence. Cutaneous anthrax commonly occurs on exposed surfaces of the arms or hands, and sometimes on the face and neck.

A 54-year-old male presents to the ER with a sore on his right shoulder. Physical examination demonstrates an ulcer with central black eschar surrounded by edema. Exudate microscopy reveals chains of large Gram-positive rods. Which of the following would be most helpful in making the diagnosis? A. Family history B. Occupation C. Recent contact with foreigners D. Allergies E. Current medications F. Sexual history G. Prior ER visits

*The answer is D.* Penicillin remains the first-line drug of choice for pneumococcal pneumonia, except in patients with penicillin allergy and in the relatively few areas in which pneumococcal strains with high-level penicillin resistance exist. Alternative therapies include erythromycin and vancomycin. The third-generation cephalosporin cefotaxime (choice A) is not usually used for pneumococcal pneumonia. Chloramphenicol (choice B) is not usually used for pneumococcal pneumonia. Erythromycin (choice C) is a good alternative therapy for pneumococcal pneumonia, but is usually used only when a penicillin allergy is present. Vancomycin (choice E) is not the first-line therapy, but it is a good alternative in patients allergic to penicillin or when high-level penicillin resistance (relatively uncommon) is present.

A 54-year-old woman suffering from influenza deteriorates and develops shaking chills and a high fever. Physical examination is remarkable for dullness to percussion at the left base and decreased breath sounds on the left. Chest x-ray confirms the diagnosis of lobar pneumonia, presumed to be caused by Streptococcus pneumoniae. The patient has no known drug allergies. Which of the following antibiotics would be most appropriate to treat the patient's condition? A. Cefotaxime B. Chloramphenicol C. Erythromycin D. Penicillin E. Vancomycin

*The answer is B.* Enterococcus faecium is most likely to be vancomycin or multiply drug-resistant. The other organisms are sensitive to vancomycin.

A 55-year-old man was admitted to a local hospital with fever and chills. The patient was human immunodeficiency virus positive and had received multiple courses of antibiotics. Blood cultures grew gram-positive cocci, which tested positive with group D streptococcal anti -sera. The isolate was resistant to penicillin and vancomycin. Which one of the following is the most likely pathogen? A. Streptococcus pneumoniae B. Enterococcus faecium C. Streptococcus pyogenes D. Streptococcus agalactiae E. Streptococcus mutans

*The answer is B.* The patient is probably suffering from bacterial endocarditis caused by S. epidermidis infection of the prosthetic heart valve. S. epidermidis is a coagulase-negative organism that is unable to ferment mannitol and is sensitive to novobiocin but usually resistant to penicillin. Patients with congenital heart malformations, acquired valvular defects (for example, rheumatic heart disease), prosthetic valves, and previous bacterial endocarditis show an increased incidence of bacterial endocarditis. Intravenous drug users also have a high risk for infection. S. pneumoniae and S. agalactiae can be ruled out, because streptococci are catalase negative, which is a feature that distinguishes them from catalase-positive staphyococci.

A 57-year-old man arrives at the emergency room complaining of weakness, fatigue, and intermittent fever that has recurred for several weeks. The patient had a cardiac valvular prosthesis implanted 5 years earlier. Physical examination reveals petechiae (pinpoint, nonraised, purplish red spots caused by intradermal hemorrhage) on the chest and stomach. Blood cultures grew catalase-positive, coagulase-negative, cocci. The gram-positive organisms failed to ferment mannitol, and their growth was inhibited by novobiocin. What is the most likely infectious agent? A. Staphylococcus aureus B. Staphylococcus epidermidis C. Staphylococcus saprophyticus D. Streptococcus pneumoniae E. Streptococcus agalactiae

*The answer is E.* The disease that this child has is diphtheria, which is caused by Corynebacterium diphtheriae. Bacillus anthracis causes the disease anthrax and there is no leathery membrane structure produced in the pharynx in anthrax. None of the Clostridium species listed above (botulinum, perfringens, or tetani) produce a leathery membrane in the pharynx, and besides they are all strict anaerobes and would not grow in the oral cavity. C. Diphtheriae does produce a leathery membrane in the oral cavity due to its production of the diphtheria toxin whose mechanism of action is inhibition of protein synthesis by inactivation of elongation factor 2, which is a mammalian protein that transfers the amino acid from the t-RNA to the growing polypeptide chain .

A 6-year-old girl in Russia developed a sore throat and was taken to the doctor by her parents. The doctor diagnosed a "strep throat" and gave her a shot of penicillin. The penicillin shot did not help, and the child's health worsened and she was brought back to the doctor. Now the child complained of more than a sore throat. Now she refused to eat and was very lethargic. She also had a fever of 40°C when the doctor reexamined the child, he observed a normal chest sound, a productive pharyngitis, and inflamed cervical lymph nodes. A throat culture did not reveal any Group A streptococci, and the child was becoming increasingly lethargic. The doctor then noticed a structure in the back of the child's throat that looked like a leather membrane. The parents told the doctor when he asked that the girl had received no vaccinations. The doctor then knew what disease he was observing. The organism most likely to be the causative agent of this infection was which of the following? a. Bacillus anthracis b. Clostridium botulinum c. Clostridium perfringens d. Clostridium tetani e. Corynebacterium diphtheriae

*The answer is C.* Diphtheria is caused by Corynebacteria diphtheriae. Acute infection of the naso- and oropharynx causes pseudomembranous pharyngitis. The diphtheria exotoxin, an AB exotoxin specific for neural and cardiac tissue causes CNS and cardiac sequelae in some patients as well. This toxin ribosylates and deactivates elongation factor- 2, thus inhibiting human protein synthesis. Clinical signs and symptoms of diphtheria infection include sore throat, fever lymphadenopathy, upper airway dyspnea, and odynophagia. The disease is transmitted via respiratory droplets. Treatment of an acute C. diphtheriae infection requires (in order of importance): 1. administration of diphtheria antitoxin, 2. administration of penicillin or erythromycin, and 3. administration of the DPT vaccine. The diphtheria antitoxin inactivates all circulating toxin, but is ineffective against toxin that has already gained access to cardiac or neural cells. Thus rapid administration of antitoxin is essential. Antibiotic therapy kills the bacteria, halting release of new exotoxin into the bloodstream and preventing disease transmission. Immunization with the DPT vaccine provides lasting immunity against future diphtheria infection.

A 6-year-old immigrant from Eastern Europe is brought to the emergency room with difficulty breathing and a low- grade fever. Physical examination reveals neck swelling, palatal paralysis, and a gray pharyngeal exudate. The parents are unable to provide information regarding the child's vaccination history. Which of the following interventions is most likely to improve this patient's prognosis? A. Antibiotic therapy B. Active immunization C. Passive immunization D. Adequate hydration E. Anti-inflammatory medications

*The answer is D.* Corynebacteria (koryne, club) are Gram positive catalase-positive, aerobic or facultatively anaerobic club-shaped rods. The genus is composed of the species Corynebacterium diphtheriae and the non diphtheria corynebacteria, also known as diphtheroids. (Once thought to be clinically insignificant diphtheroids are now recognized as pathogens that cause diseases like septicemia, urinary tract infections erythrasma and endocarditis in immunocompromised hosts.) C. diphtheriae will grow on cysteine-tellurite agar as dark black slightly iridescent colonies. It can also be cultured in Loffler's medium where it will develop cytoplasmic metachromatic granules (visualizable after staining with an aniline dye such as methylene blue). Because culturing the organism may take days and because diphtheria has high mortality that warrants immediate treatment more rapid diagnostic mechanisms such as the immunochromatographic strip assay are being developed. *Educational Objective:* C. diphtheriae is cultured on cysteine-tellurite agar. The resultant colonies are black in color. The bacterium produces intracellular polyphosphate granules called metachromatic granules that can be detected on microscopy after methylene blue staining.

A 6-year-old male is brought to the pediatric emergency room with fever and sore throat. The parents tell you that the child has not received any immunizations. Physical exam reveals a grey pharyngeal exudate and Gram stain shows scant Gram positive organisms. Which of the following culture types would facilitate the growth of this organism? A. MacConkey agar B. Thayer-Martin VON medium C. Blood agar containing bile and hypertonic saline D. Cysteine-tellurite agar E. Bordet-Gengou medium

*The answer is C.* The coagulase-negative staphylococci, particularly Staphylococcus epidermidis, have been identified as a major cause of infections in patients with predisposing factors such as indwelling catheters or implanted foreign bodies. These bacteria have the ability to colonize intravenous catheters and prosthetic devices (heart valves, vascular grafts. peritoneal dialysis catheters) because of their ability to produce a polysaccharide slime which allows adherence to the prosthetic devices. When reported, it is initially difficulty to differentiate culture contamination from real infection because S. epidermidis is a part of the normal skin flora but recovery of organism from multiple blood cultures indicates infection. These organisms can be resistant to multiple antibiotics and initial aggressive antibiotic treatment is a must pending antibiotic sensitivity analysis. S. epidermidis isolates are susceptible in vitro to vancomycin and rifampin and are frequently resistant to methicillin; therefore vancomycin combined with rifampin or gentamicin or both is recommended for therapy of serious infections caused by methicillin resistant strains. S. epidermidis can cause an indolent endocarditis after valve replacement (compared to the aggressive endocarditis associated with S. aureus in IV drug abusers) but if left untreated it can lead to intracardiac abscess formation dehiscence of the prosthetic valve from the heart, and septic embolization.

A 62-year-old Caucasian male who recently underwent a mitral valve replacement is having low-grade fevers. He also complains of dyspnea and malaise. Repeated blood cultures grow Gram-positive cocci in clusters that are catalase-positive and coagulase-negative. Which of the following is the best initial treatment for this patient? A. Penicillin 0 B. Nafcillin C. Vancomycin D. Ciprofloxacin E. Erythromycin F. Ceftriaxone

*The answer is C.* Antibiotics disturb the normal intestinal microbial ora and can lead to overgrowth of Clostridium difficile, an organism that is sometimes part of the normal gut flora or that gains access to the patient after antibiotics have killed most of the commensal bowel flora. C. difficile tends to survive the antibiotic therapy that kills most of the other bacteria that compose the normal gut flora. These other organisms normally are responsible for keeping potential pathogens like C. difficile in check. In the absence of other intestinal inhabitants, the vegetative cells of C. duff/die produce enterotoxin A, which causes watery diarrhea, and cytotoxin B, which causes colonic epithelial cell necrosis and stimulation of fibrin deposition. C. difficile disease may range in severity from mild transient watery diarrhea to severe pseudomembranous colitis. Metronidazole is the treatment of choice for C. difficile pseudomembranous colitis.

A 65-year-old male hospitalized for community-acquired pneumonia develops fever and loose stools on his fourth hospital day. He recently traveled to Mexico for vacation without taking any prophylactic antibiotics or receiving any vaccinations. Sigmoidoscopy demonstrates white yellow membrane-like plaques on the colonic mucosa and biopsy shows that these plaques are composed of fibrin and inflammatory cells. The patient's condition is most likely associated with which of the following? A. Recent travel to Mexico B. Missed vaccination C. Antibiotic therapy D. Gastritis prophylaxis E. Consumption of home-canned foods

*The answer is A.* The most common cause of community acquired pneumonia in this age group is Streptococcus pneumoniae. The X-ray and microbiological findings are most consistent with a diagnosis of pneumococcal pneumonia. Following treatment, this patient should be advised to be vaccinated with the 23-valent pneumococcal vaccine. Streptococcus pyogenes does not typically present as pneumonia. Streptococcus agalactiae generally afflicts neonates. The enterococci (Enterococcus faecium and Enterococcus faecalis) do not exhibit the Quellung reaction and do not present as community acquired pneumonia.

A 65-year-old male presents to his family physician with a rapid onset fever, chest pain and cough productive of rusty-yellow sputum. Chest X-ray shows focal lobar infiltrates. A Gram stain of a sputum sample contained many polymorphonuclear leukocytes and extracellular gram-positive diplococci. Capsule-specific antibodies bound to the diplococci resulted in a positive Quellung reaction. Which of the following is the most likely pathogen? A. Streptococcus pneumoniae B. Enterococcus faecium C. Streptococcus pyogenes D. Streptococcus agalactiae E. Enterococcus faecalis

*The answer is B.* There are two main virulence factors of B. anthracis: 1. the anthrax exotoxin composed of edema factor, lethal factor, and protective antigen that are required for either of the aforementioned toxins to function and 2. the antiphagocytic polygamma-D-glutamic acid capsule. Edema factor is a calmodulin dependent adenylate cyclase that causes an increase in the cAMP concentration inside host cells. Edema factor is though to play an important role in suppression of neutrophil function and accumulation of fluid within and between cells leading to edema. Bordetella pertussis produces an edema factor-like exotoxin simply called "extracellular adenylate cyclase." The Bordetella pertussis adenylate cyclase toxin is very important for successful colonization of the organism in the respiratory tract. The extracellular adenylate cyclase secreted by B. pertussis is able to enter eukaryotic cells and cause a dramatic increase in cAMP levels leading to neutrophil dysfunction just as edema factor in B. anthracis does.

A 65-year-old male presents to the ER with a sore on his right hand. He is involved in the wool business and has recently returned from a tour of wool processing plants in a series of Asian countries. Physical examination reveals an ulcer with black eschar and significant surrounding edema. The toxin that caused edema around the ulcer in this patient has an action most similar to that of a toxin produced by: A. Streptococcus pyogenes B. Bordetella pertussis C. Shigella dysenteriae D. Clostridium botulinum E. Clostridium difficile

*The answer is B.* C. difficile colonizes the colonic mucosa and releases toxins that cause mucosal inflammation, cell damage and ultimately cell death with mucosal necrosis. This organism is present in 2-3% of healthy adults and in about 70% of healthy infants as a part of the gut's normal microbial flora. Disease results when the patient uses antibiotics that kill a large portion of the commensal organisms in the gut (which normally keep potentially pathogenic organisms such as C. difficile in check). Pathogenic strains of C. difficile produce 2 distinct toxins: toxin A (enterotoxin) and toxin B (cytotoxin). Both play a role in the pathogenesis of C. difficile colitis. The toxins bind specific receptors on intestinal mucosal cells and are internalized where they exert their intracellular effects. Toxin A acts as a neutrophil chemoattractant leading to mucosal inflammation, loss of water into the gut lumen (producing diarrhea), and mucosal death. Toxin B causes actin depolymerization, loss of cellular cytoskeleton integrity, cell death and mucosal necrosis.

A 65-year-old male taking amoxicillin for sinusitis is hospitalized with diarrhea and abdominal cramps. Complete blood count shows leukocytosis. This toxin responsible for his current condition primarily damages which of the following components of intestinal mucosal cells? A. Mitochondrial energy production B. Cytoskeleton integrin C. Apical ion transport D. Ribosomal protein synthesis E. Cell membrane integrity

*The answer is E.* Enterococcal endocarditis usually occurs in elderly men who have recently undergone manipulation of areas colonized by this organism, such as the GI or GU tracts. In women enterococcal endocarditis can occur following obstetrical procedures. Enterococci are Lancefield Group D streptococci, Gram positive cocci that can be cultured in hypertonic (6.5%) saline and bile. These bacteria are gamma-hemolytic. The enterococci are resistant to many antibiotics and can be very difficult to treat. Penicillin is often combined with an aminoglycoside for a synergistic effect, but increasingly bacteria resistant to both aminoglycosides and β-lactams (including penicillinase-resistant types) are emerging. Additionally, vancomycin-resistant and linezolid-resistant enterococci have emerged.

A 67-year-old male is hospitalized with low-grade fevers fatigue and a diastolic murmur at the left sternal border. Blood cultures reveal Gram positive cocci that are catalase-negative and able to grow in 6.5% saline. This patient's medical history is most likely to reveal which of the following procedures in the past month? A. Dental extraction B. Skin biopsy C. Sinus drainage D. Nasal polypectomy E. Cystoscopy

*The answer is D.* This man is suffering from tetanus, caused by the exotoxin produced by Clostridium tetani. This is a ubiquitous organism found widely in the soil. Although most people are immunized against tetanus, immunity is not lifelong and requires periodic booster immunizations. Tetanospasmin produced by C. tetani prevents the release of inhibitory neurotransmitters by cleaving synaptobrevins in synaptic vesicles. The effect of this toxin is a systemic, spastic paralysis that can result in death if not aggressively treated with antitoxin.

A 67-year-old man, who is an avid gardener, presents at an emergency department. He is suffering from spastic paralysis, which began in his right hand and now extends to his jaw muscles. The causative agent of this infection produces a virulence factor with which of the following activities? A. Overstimulation of T cells with resulting massive release of cytokines B. Blockage of the release of the neurotransmitter acetylcholine from synaptic vesicles C. Adenosine diphosphate-ribosylation of EF-2, resulting in inhibition of protein synthesis D. Blockage of the release of the inhibitory neurotransmitter glycine E. Glucosylation of Rho-family GTPases

*The answer is D.* This is cellulitis caused by group A Streptococcus, or Streptococcus pyogenes. Streptokinase lyses clots, enabling the bacteria to spread in the tissues. None of the other virulence factors listed are associated with S. pyogenes. A-B exotoxins are made by a variety of toxigenic bacteria such as Clostridium botulinum or Corynebacterium diphtheriae. Lipopolysaccharide is a component of the outer membrane of Gram-negative bacteria. Uropathogenic strains of E. coli express P fimbriae. Yops proteins are virulence factors of Yersinia pestis.

A 67-year-old moderately obese man presents with fever and an erythematous, swollen, tender, warm calf. He is diagnosed clinically with cellulitis and catalase-negative, β-hemolytic Gram-positive cocci are isolated from blood cultures. Which virulence factor of the causative organism enabled it to cause this invasive disease? (A) An A-B exotoxin (B) Lipopolysaccharide (C) P fimbriae (D) Streptokinase (E) Yops proteins

*The answer is C.* Staphylococcus aureus and Streptococcus pyogenes(so called pyogenic cocci) cause prima diseases of the skin and subcutaneous tissues, frequently infecting wounds, abrasions and burns, impetigo, a vesicular, blistering eruption eventually leading to formation of a golden yellow crust, is usually seen in children and newborns, frequently occurs periorally, and can be caused by either Staphylococcus aureus and/or Streptococcus pyogenes. Acute rheumatic fever (ARE) and Acute Post Streptococcal Glomerulonephritis (APSGN) are the late sequelae of untreated or partially treated S. pyogenes (Group A Strep) infections. APSGN can follow a streptococcal skin infection (impetigo) or a streptococcal pharyngeal infection. If APSGN occurs, it will occur 1-5 weeks after the onset of the streptococcal infection. Rheumatic fever, on the other hand, is associated only with Group A Streptococcal (GAS) throat infection. Hypertension hematuria, nephritic range proteinuria, and RBC casts in the urine following a GAS infection suggest acute post streptococcal glomerulonephritis. Patients frequently present with facial edema and dark colored (cola colored) urine (Choice C). The renal damage is due to immune complex deposition on the glomerular basement membrane and activation of complement. Complement activation is responsible for the massive inflammatory response and glomerular basement membrane structural damage seen in APSGN.

A 7-year-old Caucasian boy is brought to your office with blisters on his face. Some of the blisters have broken and are covered with golden yellow crusts. Exudate microscopy reveals Gram-positive cocci in chains. Which of the following would be a component of the clinical syndrome that may follow such an infection? A. Joint pain and eye redness B. Fatigue and heart murmurs C. Face puffiness and dark urine D. Bilateral facial nerve palsy E. Abdominal pain and jaundice

*The answer is D.* Pseudomonas aeruginosa causes malignant otitis externa, which is a severe necrotizing infection of the external ear canal. Infection tends to spread to the mastoid bone, temporal bone, sigmoid sinus, base of the skull, meninges, and brain. Patients at increased risk include the elderly, those with diabetes, and the immunocompromised. Pseudomonas is also associated with many other clinical syndromes, including infection following traumatic wounds to the feet in persons wearing sneakers or rubber-soled shoes, since the organism can be cultured from the shoes. Hemophilus influenzae(choice A) produces a variety of clinical syndromes. H.Influenzae is the third-most common cause of meningitis in children aged 1 month to 18 years. It is the most common cause of acute epiglottitis, the most common cause of purulent bacterial conjunctivitis, and the second-most common cause of otitis media. Patients at risk include those with COPD and cystic fibrosis, alcoholics, splenectomized patients, and young patients. Klebsiella pneumoniae(choice B) is a gram-negative organism that produces a necrotizing pneumonia in diabetics and alcoholics. Patients typically present with an abrupt onset of fever, shaking chills, and purulent, foul-smelling sputum. Mucor (choice C) is a fungal infection that is particularly severe in the diabetic or the immunocompromised patient. In the acidotic diabetic, the fungus produces a life-threatening, invasive rhinocerebral infection. The infection begins in the nasal passages, extends into the paranasal sinuses, and spreads through the cribiform plate to the frontal lobes of the brain. Patients typically complain of headache, facial pain, and orbital swelling. Streptococcus pyogenes(choice E) causes bacterial pharyngitis, otitis media, and sinusitis. It is also associated with toxin-related diseases and skin infections.

A 73-year-old woman with a history of diabetes presents with left ear pain and drainage of pus from the ear canal. She has swelling and tenderness over the left mastoid bone. Which of the following microorganisms is the most likely causative agent? A. Hemophilus influenzae B. Klebsiella pneumoniae C. Mucor sp D. Pseudomonas aeruginosa E. Streptococcus pyogenes

*The answer is C.* This patient's presenting signs and symptoms and report of similarly ill household children are consistent with influenza infection. Outbreaks of influenza A can affect 50-75% of school-aged children, many of whom then spread the virus to family members. Individuals infected with influenza A tend to experience abrupt onset fever, headache myalgia, and malaise: signs and symptoms gradually improve over a period of two to five days. A subset of patients stricken by influenza go on to develop secondary bacterial pneumonia characterized by recurrent fever, dyspnea, and productive cough. The elderly are particularly at risk for this complication. Physical examination and chest radiograph demonstrate pulmonary consolidation. Patients recently infected with influenza are vulnerable to secondary bacterial infection because of virally-induced damage to the mucociliary clearance mechanisms of the respiratory epithelium. In order, the pathogens most often responsible for secondary bacterial pneumonia are Streptococcus pneumoniae, Staphylococcus mucus, and Haemophilus influenzae.

A 74-year-old previously healthy Caucasian male comes to his physician's office complaining of abrupt on set fever, headache, myalgias, malaise, cough and throat pain. His two granddaughters missed several days of school because of similar symptoms. Examination demonstrates mild hyperemia of the throat without any exudate, and the patient is sent home on conservative management. Five days later, he is admitted to the hospital with progressive dyspnea, chest pain, and productive cough. Which of the following pathogens is most likely to be isolated from this patient's sputum? A. Listeria monocytogenes B. Klebsiella pneumoniae C. Staphylococcus aureus D. Nontuberculous mycobacteria E. Cytomegalovirus

*The answer is B.* Clostridium difficile has been shown to be the major causative agent of pseudomembranous colitis, which causes diarrhea that most commonly starts after 3 to 4 days of antibiotic administration.

A 75-year-old patient develops diarrhea 5 days after starting antibiotic treatment for a serious staphylococcal infection. What is the most likely causative agent? (A) Clostridium perfringens (B) Clostridium difficile (C) Pseudomonas aeruginosa (D) Shigella sonnei

*The answer is A.* The other mechanisms listed account for resistance to various antibiotics in other bacterial species.

A 78-year-old man developed endocarditis due to a vancomycin-resistant strain of Enterococcus faecalis. What is the molecular basis of resistance of this organism to vancomycin? (A) A change in the terminal pentapeptide from D-ala-D-ala to D-ala-D-lac (B) A mutation in the penicillin-binding protein (C) A mutation in the porin protein (D) The acquisition of an efflux pump (E) The secretion of β-lactamase

*The answer is B.* There are more than 80 serotypes of S. pneumoniae based on variants of the capsular polysaccharide. The diversity of serotypes makes vaccine development a complex task. In adults, S. pneumoniae types 1-8 are responsible about 75% of the time. The 23-valentpneumococcal polysaccharide single-dose vaccine is recommended for all adults over age 65 and for other patients at high risk for pneumococcal sepsis (e.g. HIV patients asplenic patients, chronic obstructive pulmonary disease patients, and immunosuppressed patients). It is 50-90% efficacious (Choice B). There is also a 7-valent conjugated vaccine available for use in children less than 2-years-old, which is about 90% efficacious in these patients. The conjugated vaccine contains polysaccharide antigens that are protein-coupled in order to stimulate the T-cell dependent immune (memory) response. The N. meningitidis and H. influenzae vaccines are other examples of polysaccharide vaccines. *Educational Objective:* The pneumococcal polysaccharide vaccine is recommended for all adults over 65 years of age and for patients with COPD, asplenia, or immunosuppression. Vaccination does not completely prevent pneumonia, as this vaccine only contains antigen from 23 of the more than 80 different capsular serotypes known. The adult pneumococcal vaccine is an unconjugated polysaccharide vaccine that, unlike the infant vaccine, does not stimulate a T-helper response.

A 78-year-old nursing home resident develops high fevers, chest pain and cough productive of rusty sputum. Sputum microscopy reveals numerous lancet-shaped Gram positive cocci in pairs. The patient dies despite aggressive hydration and antibiotic treatment. A vaccine containing which of the following might have prevented this outcome? A. Recombinant surface protein B. Capsular polysaccharide C. Live attenuated bacteria D. Killed bacteria E. Inactivated toxin F. Synthetic oligopeptides

*The answer is C.* In developing countries, most cases of tetanus occur in mothers with incompletely removed placentas and in newborns with unclean and infected umbilical cord stumps. Tetanus vaccine has been available since 1925. Immunity to Clostridium tetani is produced by vaccination with a formalin-inactivated toxin, also known as toxoid. The first vaccine dose is started approximately 2 months after birth. After the initial childhood vaccine series at 2 months 4 months, 6 months 15 to 18 months and 10 to 12 years booster immunizations are required every ten years to maintain a protective level of antibody. Since lgG can cross the placenta, a newborn will achieve protection from maternally derived antibody if the mother has immunity.

A developing country is experiencing a rise in the incidence of neonatal tetanus (first 28 days after the birth). Which of the following measures would be the most appropriate prevention measure to control the problem? A. Routine vaccination at birth B. Peripartal antibiotics C. Vaccination of young adults D. Early postpartum breast feeding E. Extended postpartum hospital stay

*The answer is E.* Observation of diphtheria toxin production is required to prove the diagnosis. Items A and B are presumptive indicators. β Phage is a temperate phage, and lytic activity is not observed. Cornyebacterium diphtheriae is noninvasive, and the organism (but not the toxin) is recovered only from surface infections such as those of the oropharynx and skin lesions.

A diagnosis of diphtheria is confirmed by: A. microscopic appearance of organisms stained with methylene blue. B. isolation of a typical colony on Tinsdale agar. C. isolation of typical organisms from materials such as blood, showing invasiveness. D. detection of β phage plaques in cultures of suspicious isolates. E. demonstration of toxin production by a suspicious isolate.

*The answer is E.* Early antibiotic treatment of streptococcal pharyngitis can prevent many cases of rheumatic fever (RE). RE and RE- induced valvular heart disease is still a major problem in some developing nations. Due to poor living conditions, crowding and little access to health care, these patients acquire heart disease more rapidly and in more severe forms. The mechanism of rheumatic valvular damage is repeated stimulus of E-lymphocytes to mount an antibody response against antigens on Group A Streptococci (GAS) that are homologous to self antigens in the heart and CNS. Therefore, recurrent untreated GAS pharyngitis will lead to a more rapid onset and increased severity of rheumatic valvular disease. RE involves three organ systems: joints, heart and brain. Rheumatic (migratory) polyarthritis causes joint pain and inflammation that moves from one joint to another and typically affects children ages 5-15. The inflammation of the heart (rheumatic carditis) involves all layers of the heart from the pericardium to the endocardium and can cause chest pain in the case of pericarditis. A characteristic murmur of valvular incompetence usually involves the mitral valve and/or the aortic valve. The mitral valve dilates and chordae elongate causing the valve leaflets to incompletely close resulting in mitral insufficiency or regurgitation requiring excessive treatment and frequently surgery (Choice E). Eradication of infective streptococci from the pharynx prevents subsequent episodes of RE. The drug of choice is penicillin. No penicillin resistant strains of S. pyogenes have been detected as yet.

A group of physicians in a developing country is advocating the early empiric use of penicillin in the treatment of clinically diagnosed bacterial pharyngitis, a condition that is quite common in children of that nation. What would you expect to decrease as a result of their policy in the long term? A. Expensive antibiotic requirement B. Nephritic disease C. Deaths caused by diarrhea D. Bronchial asthma hospitalizations E. Cardiac surgeries F. Vaccination needs

*The answer is E.* The organism fi ts the description of S. aureus (catalase-positive, Gram-positive coccus). The rapid onset and severity of symptoms fit the clinical picture of MRSA. Of the other organisms listed, only group A Streptococcus (GAS) and Enterococcus are Gram-positive cocci. While GAS is β-hemolytic, it is not catalase-positive. Enterococci are rarely β-hemolytic and are also catalase-negative.

A patient injured herself at home in the garden, and after several days, the resulting skin lesion became infected and intensely painful and she sought medical help. Culture of the wound grew Gram-positive cocci that were β-hemolytic on blood agar. On nutrient agar, the organism produced effervescence when hydrogen peroxide was added to the colonies (shown in the photograph). Of the following organisms, what is the patient most likely infected with? (A) Clostridium sp. (B) Enterococcus (C) Group A Streptococcus (D) Meningococcus (E) Methicillin-resistant Staphylococcus aureus (MRSA)

*The answer is E.* This drug is commonly used for community-acquired MRSA infections. Amoxicillin/clavulanate, cephalexin, and dicloxacillin are β-lactam antibiotics used for β-lactamase-producing strains of S. aureus. The mechanism of β-lactam resistance in MRSA is due to the production of an altered penicillin-binding protein; thus, these are not appropriate antibiotics for this strain. Polymyxin B is an antifungal drug.

A patient injured herself at home in the garden, and after several days, the resulting skin lesion became infected and intensely painful and she sought medical help. Culture of the wound grew Gram-positive cocci that were β-hemolytic on blood agar. On nutrient agar, the organism produced effervescence when hydrogen peroxide was added to the colonies (shown in the photograph). Based on these initial findings, what is the best antibiotic choice for the patient in the above case? (A) Amoxicillin/clavulanate (B) Cephalexin (C) Dicloxacillin (D) Polymyxin B (E) Trimethoprim-sulfamethoxazole (TMP-SMZ)

*The answer is A.* Antibiotic treatment is often complicated by gastrointestinal disturbances, including pseudomembranous colitis. Certain drugs, including clindamycin, are more likely to cause this complication.

A predisposing factor in pseudomembranous colitis is: A. clindamycin treatment. B. neonatal age. C. diet high in dairy products. D. cholecystitis. E. older age (older than age 60 years).

*The answer is D.* The child was diagnosed with impetigo contagiosa, a pyoderma. The two most common agents are coagulase-positive staph (Staphylococcus aureus) and group A strep (Streptococcus pyogenes). Mixed infections of both organisms regularly occur, but this fact has little relevance with the current status of the child. What is relevant, however, and what distinguishes this case, is the probable glomerulonephritis as noted by the clinical presentation. Described as a rare immune-mediated disease, glomerulonephritis is precipitated by certain strains of strep A pyoderma. Glomerulonephritis can occur even if the impetigo was successfully treated.

A preschool-aged child from Georgia is hospitalized because of edema and hematuria. Of note is that the child was recently successfully treated for a pyoderma (see photo). Which microbe is likely associated with this child's condition? (A) Acid-fast Nocardia asteroides complex (B) Coagulase-negative Staphylococcus (C) Escherichia coli or related Gram-negative enteric (D) Group A Streptococcus (E) Pseudomonas aeruginosa

*The answer is B.* Pathogenic species of S. pneumoniae produce an outer polysaccharide capsule that prevents phagocytosis by the host immune system. This capsule is S. pneumoniae major virulence factor; in laboratory experiments where itis removed, the organism becomes nonpathogenic. Because the capsule is antigenic, S. pneumoniae infection elicits anti-capsule antibody production and lasting immunity against that particular strain. (Opsonization of the capsule by antibody and complement renders the organism non virulent.) Unfortunately there are 82 different capsule types due to variations in the polysaccharide, and immunity is only to the single infecting strain. The pneumococcal vaccine generates immunity against the capsular antigens of several important strains. When viewed under a microscope, the capsule swells when specific anti-capsular antibodies are added (Choice B), an effect known as the "quellung reaction". This reaction can be used to identify S. pneumoniae and to serotype the isolate.

A researcher finds that Streptococcus pneumoniae injected intraperitoneally is uniformly fatal to mice. Antibodies with which of the following in vitro effects on the bacterium would prevent this outcome? A. Decreased motility B. Capsular swelling C. Inhibited hemolysis D. Loss of cytotoxicity E. Impaired adhesion

*The answer is D.* All of the organisms listed can cause infection in infants, but Listeria monocytogenes is the one to cause potentially fatal granulomas (granulomatis infantiseptica) following in utero infection. Clostridium botulinum(choice A) can cause infant botulism, characterized by failure to thrive, and progressive muscular weakness. Escherichia coli(choice B) can cause diarrhea, pneumonia, and meningitis in infants. Haemophilus influenzae(choice C) can cause acute epiglottitis, meningitis, pneumonia, and otitis media in young children. Neisseria gonorrhoeae(choice E) can be transmitted via the birth canal, and causes ophthalmia neonatorum.

A very ill neonate has widespread granulomas. In utero infection with which of the following organisms is suggested by this finding? A. Clostridium botulinum B. Escherichia coli C. Haemophilus influenzae D. Listeria monocytogenes E. Neisseria gonorrhoeae

*The answer is D.* The boy met his unfortunate end for a variety of different reasons. The primary reason the boy died was because he had not recently received a tetanus booster. If he had told his parents about the nail puncture they could have taken him to the doctor to get a tetanus shot. That shot would have saved his life. Now we will go to the microbiology of the question. The boy came down with tetanus because Clostridium tetani is a spore former, and because the spores can be found in the soil, they can also be found on rusty nails. The rust on the nail does not play a role in this question at all. The second important thing about Clostridium tetani is that it is a strict anaerobe. So the nail drives the spores into the boy's foot and causes tissue damage (necrosis), and where there is necrosis there is no blood supply, and where there is no blood supply, there is no oxygen, and where there is no oxygen there are anaerobic conditions. Therefore, the Clostridium tetani spores can germinate (crack open) and allow the vegetative form of the organism to come out, and it is the vegetative form that produces the potent heat-labile neurotoxin. The Clostridium tetani neurotoxin blocks release of the neurotransmitters for inhibitory synapses, thus causing the involuntary muscle spasms and respiratory failure.

A young boy, 9 years of age, is outside playing in the summer in Texas and steps on a board with a rusty nail in it. The nail goes right through his gym shoe and enters his right foot. He does not tell his parents about it because he is sure his mother will yell at him because she is always telling him not to do what he just did. He is also afraid she will curtail his playing outside privileges. Besides, it did not bleed much and the bleeding stopped before he went in for supper. Besides, the boy's family did not believe in going to the doctor for every little thing. In fact, the boy cannot remember the last time he saw a doctor. About a week later, the boy developed a sore throat and then 4 days later his parents did take him to the hospital (reluctantly) with difficulty in swallowing, talking, and breathing. Also, the boy began to experience muscle spasms. The hospital doctor recognized the signs of tetanus and immediately administered tetanus immune globulin. The doctor asked the parents when the last time the boy had received a tetanus shot and they replied that they did not know. After being in the hospital for a week, the boy, unfortunately, died of respiratory failure. This unfortunate incident occurred because which of the following facts represents the best answer? a. The causative organism, Clostridium tetani, is a strict anaerobe b. The causative organism, Clostridium tetani, is a strict aerobe c. The causative organism, Clostridium tetani, produces a potent heat-labile neurotoxin d. The causative organism, Clostridium tetani, is a spore producer, a strict anaerobe, and produces a potent heat-labile neurotoxin e. The causative organism, Clostridium tetani, is a spore former, a strict aerobe, and produces a potent heat-labile neurotoxin

*The answer is E.* Exfoliatin is a virulence factor, produced by some Staphylococcus aureus strains, cleaves desmosomes, resulting in loss of the outer layers of skin. This manifestation is also known as scalded skin syndrome. The toxic shock syndrome toxin is a superantigen produced by some S. aureus strains. This toxin causes systemic effects and has been associated with tampon use. Panton-Valentine Leukocidin is a hemolysin that lyses white blood cells and is produced by many community-acquired MRSA strains. Protein A is a virulence factor that allows S. aureus to evade an immune response by binding the Fc region of IgG, resulting in the inverse orientation of the antibody. Thus, the antibody cannot effectively opsonize the bacterium. The thin microcapsule of S. aureus is also associated with immune evasion.

An 18-month-old child was brought the pediatrician's office with what appeared to be a sunburn, although the parents denied that the child had been over exposed to the sun. The parents did recall seeing an area of redness and small blisters on the child's arm the night before. Which of the following virulence factors is critical to this disease manifestation? A. Toxic shock syndrome toxin. B. Panton-Valentine Leukocidin C. Protein A D. Capsule E. Exfoliatin

*The answer is B.* The patient described in this question exhibits a clinical picture consistent with acute rheumatic fever as well as long term sequelae of rheumatic fever. Rheumatic fever is a syndrome of fever arthritis, subcutaneous nodules characteristic rash (erythema marginatum), involuntary rhythmic movements of the extremities (Sydenham chorea) and myocarditis leading to valvular in sufficiency of the mitral or aortic valves. It follows untreated Group A Streptococcal (GAS) pharyngitis. The symptoms of rheumatic fever result from structural homology between antigenic determinants. *Educational Objective:* Rheumatic fever is an autoimmune reaction that occurs following untreated Streptococcus pyogenes (GAS) pharyngitis. Antigenic similarity between bacterial antigens and normal "self" antigens in the heart and CNS are believed to cause formation of anti-self antibodies resulting in RE.

An 8-year-old Caucasian boy is brought to your office with throat pain fever and malaise. Physical examination reveals white exudates on his tonsils and swollen anterior cervical lymph nodes. If the boy returns in a month with fatigue joint pain and chest pain and later in life he develops a heart murmur, which of the following would be the most likely explanation for his condition? A. Protein A-mediated opsonization block B. Bacterial and human epitope homology C. Immune complex deposition D. Exotoxin-induced T-cell receptor activation E. Coronary artery aneurysm formation

*The answer is B.* The infection described is strep throat caused by Streptococcus pyogenes, also referred to as group A Streptococcus. A potential complication of some strains is rheumatic fever, the manifestations of which include carditis, arthritis, and Sydenham chorea. Antibiotic treatment is recommended to prevent this complication. S. pyogenes is also associated with skin infections and necrotizing fasciitis. Strep throat very rarely leads to bacteremia. The organism is not associated with encephalitis, hepatitis, or meningitis.

An 8-year-old girl presents to the urgent care facility with 1 day history of fever, sore throat, malaise, and headache. Physical examination reveals a temperature of 39 °C, cervical lymphadenopathy, and a whitish tonsillar exudate. A rapid diagnostic test is negative; however, culture of the exudate grew Gram-positive cocci in chains that caused β-hemolysis on blood agar. What complication is prevented by appropriate antibiotic therapy for this infection? (A) Bacteremia (B) Carditis (C) Encephalitis (D) Hepatitis (E) Meningitis

*The answer is E.* There are over 100 serotypes of Streptococcus pyogenes which vary in their pathogenic potential. Certain strains are more invasive than others, and rheumatogenic strains are associated with acute rheumatic fever. Serotypes are based on antigenic differences in the M protein, a coiled, dimeric protein that functions as an important virulence factor. It is coded for by the bacterial emm family of genes. The N terminus of the M protein dimer extends above the cell wall and is highly variable while the carboxyl terminus, anchored in the cytoplasmic membrane, is highly conserved between strains. The M protein functions in adhesion and has antiphagocytic properties. Some strains of S. pyogenes have a capsule. Capsular antigens distinguish virulent strains of S. pneumoniae from less virulent strains; however, strains are not distinguished from one another by variation in capsular antigens. Cell wall antigens are the basis of Lancefi eld classification of streptococcal species. This organism lacks flagella as well as O antigens, which are part of the structure of lipopolysacccharide, a component of Gram-negative cell walls.

An 8-year-old girl presents to the urgent care facility with 1 day history of fever, sore throat, malaise, and headache. Physical examination reveals a temperature of 39 °C, cervical lymphadenopathy, and a whitish tonsillar exudate. A rapid diagnostic test is negative; however, culture of the exudate grew Gram-positive cocci in chains that caused β-hemolysis on blood agar. What is the basis of serotyping of the pathogen in the above question? (A) Capsular antigens (B) Cell wall antigens (C) Flagellar antigens (D) O antigens (E) M antigens

*The answer is C.* Of the enteric pathogens listed in the answer choices, only Campylobacter species can be transmitted from domestic animals to humans. Campylobacter is a curved Gram negative rod with a filament that allows it to move in a characteristic "corkscrew" fashion. Campylobacter jejuni is the most common cause of acute gastroenteritis in children and adults in industrialized countries. Transmission is via the fecal-oral route. The organism can be acquired from: 1. Domestic animals, such as cattle, sheep, dogs, and chickens. This route of transmission is common in farm and laboratory workers. 2. Contaminated food, such as undercooked poultry and unpasteurized milk. Campylobacter species cause inflammatory diarrhea (initially watery, later bloody), accompanied by abdominal cramping, tenesmus and leukocytes in stool. The abdominal pain may mimic appendicitis. Campylobacter is the most common infectious agent associated with Guillain-Barré syndrome.

An 8-year-old male is brought to the ER with a two day history of fever, abdominal pain and diarrhea. Careful history taking reveals that the patient's pet puppy had diarrhea one week ago. Aside from the present episode, the patient has no significant past medical history and all of his vaccinations are up-to-date. His stool is negative for ova and parasites. Which of the following is the most likely cause of this patient's symptoms? A. Staphylococcus aureus B. Shigella C. Campylobacter D. Bacillus cereus E. Vibrio parahemolyticus F. Giardia lamblia

*The answer is B.* Protein M is the major virulence factor for Streptococcus pyogenes; it inhibits phagocytosis and the activation of complement. The cell surface of Streptococcus pyogenes is biochemically diverse: Lancefield Group A Streptococci are divided into more than 80 serogroups based on M proteins. In addition to their antiphagocytic and anticomplement properties M protein is also cytotoxic for neutrophils in the serum and a mediator of bacterial

Group A Streptococci demonstrate significant resistance to phagocytic killing when placed in fresh human blood. This resistance can be most effectively overcome by adding antibodies to which of the following? A. Hyaluronate B. Protein M C. Streptolysin O D. DNase E. Streptokinase F. Teichoic acid

*The answer is D.* Over 400 types of bacteria inhabit the healthy human gastrointestinal tract as part of the normal gut flora. In healthy humans, the gastrointestinal tract has very few aerobes (i.e. Pseudomonas), approximately 10% facultative anaerobes (Escherichia coli, Klebsiella, Streptococcus Lactobacillus, Staphylococcus, and Bacillus), and a majority of strict anaerobes (Bifidobacterium, Fusobacterium, Clostridium, Eubacterium Peptococcus, and Peptostreptococcus). These intestinal bacteria effectively suppress overgrowth of Clostridium difficile and many other potentially pathogenic bacteria by competing for nutrients and adhesion sites within the gut. Treatment with antibiotics can alter the intestinal balance of bacteria leading to a potential overgrowth of pathogenic strains and clinical disease. C. difficile requires prior antibiotic use to establish infection. It causes disease by releasing two toxins that damage the mucosal lining of the large intestine leading to diarrhea (Toxin A) and necrosis (Toxin B) with pseudomembrane formation.

Healthy volunteers orally inoculated with pathogenic strains of Clostridium difficile do not develop signs of infection. Which of the following protective mechanisms is most likely responsible? A. Cell-mediated immunity B. Mucosal IgA antibodies C. Gastric acidity D. Intestinal biomass E. Rapid gastrointestinal transit

*The answer is D.* C. tetani can be transmitted by the penetration of skin by a rusted nail.

How could you get clostridium tetani? A. By eating smoked fish or improperly canned foods at home B. Infection through infected animal or soil C. Improperly cooking food, at lower temperatures. D. Penetration of skin by a rusted nail E. Innoculated with trauma into muslcles

*The answer is A.* You can get Bacillus Cereus by improperly cooking food at lower temperatures.

How would you get Bacillus Cereus? A. Improperly cooking food, at lower temperatures. B. Infection through infected animal or soil C. By eating smoked fish or improperly canned foods at home D. Innoculated with trauma into muscles E. Penetration of skin by a rusted nail

*The answer is B.* Certain strains of S. pneumoniae express capsular polysaccharides that inhibit phagocytosis, making it a successful pathogen. Strains lacking the capsule are not pathogenic. S. pneumoniae is able to undergo transformation, a process involving uptake and expression of chromosomal fragments from the environment, when neighboring bacteria die and lyse (Choice B). Bacteria capable of taking up free DNA, that is, capable of undergoing transformation, are said to be 'competent.' Streptococcus pneumoniae, Haemophilus influenza and Neisseria meningitidis are bacteria that have this ability. Through this process, non-virulent, non-capsule forming strains of S. pneumoniae can acquire the genetic material that codes for the capsule and thus gain virulence.

In a research experiment, Strain of Streptococcus pneumoniae produces no effects when injected into the mouse peritoneal cavity. However, when grown on media alongside a virulent strain of S. pneumoniae (Strain 2), Strain acquires the ability to cause death. Which of the following processes most likely accounts for this acquisition of virulence? A. Pilus-mediated transfer B. Chromosomal fragment uptake from media C. Phage-mediated DNA transfer D. Spontaneous mutation E. Transposon-mediated DNA transfer

*The answer is A.* Several species of Clostridium have been linked to gas gangrene; however, Clostridium perfringens is the most common. Clostridia are anaerobes, emit strong odors—often described as fetid—and are gas producers in tissue or when grown in laboratory media. They produce endospores when challenged environmentally. The body reacts vigorously (sometimes to its detriment) to the huge array of exotoxins produced during an infection. These tissue-damaging toxins are virulence factors that allow the bacteria to "set up housekeeping" in the host. α-Toxin, a lecithinase that produces tissue necrosis, is of paramount importance in the pathogenesis of C. perfringens. Pyrogenic exotoxin A, an erythrogenic toxin of Streptococcus pyogenes (group A streptococci), is associated with scarlet fever; streptolysin S is linked to the b-hemolysis characteristically seen with group A streptococci on sheep blood agar; enterotoxin is responsible for eliciting the symptoms associated with food poisoning by Staphylococcus aureus; and pyocyanin is a bluish pigment and a primary virulence factor common to Pseudomonas aeruginosa.

Infected pressure sores were observed on the buttock of an elderly, bedridden patient recently treated for a malignancy of the rectum. Some of the lesions are conspicuously necrotic, exceptionally painful to the touch, and readily give off a musty sweet odor. Moreover, these lesions seem to have developed overnight. A Gram stain of the watery discharge from one of the lesions reveals an abundance of bacteria (shown in the photograph). Which virulence factor is most important when considering the pathology of this case? (A) α-Toxin (lecithinase) (B) Enterotoxin (C) Pyocyanin (D) Pyrogenic exotoxin A (E) Streptolysin S

*The answer is A.* Several species of Clostridium have been linked to gas gangrene; however, Clostridium perfringens is the most common. Clostridia are anaerobes, emit strong odors—often described as fetid—and are gas producers in tissue or when grown in laboratory media. They produce endospores when challenged environmentally.

Infected pressure sores were observed on the buttock of an elderly, bedridden patient recently treated for a malignancy of the rectum. Some of the lesions are conspicuously necrotic, exceptionally painful to the touch, and readily give off a musty sweet odor. Moreover, these lesions seem to have developed overnight. A Gram stain of the watery discharge from one of the lesions reveals an abundance of bacteria (shown in the photograph). What is the most likely diagnosis? (A) Clostridial gas gangrene (B) Pseudomonas ecthyma gangrenosum (C) Staphylococcal cellulitis (D) Staphylococcal pyomyositis (E) Streptococcal necrotizing fasciitis

*The answer is C.* The cough is suggestive of infection with S. pneumoniae. All streptococci are catalase negative. In the laboratory, the optochin and bile solubility tests are used to differentiate pneumococci from S. viridans and Group D Streptococci. Alpha hemolytic organisms sensitive to optochin are identified as S. pneumoniae while alpha-hemolytic organisms that are optochin-resistant are identified as S. viridans. S. pneumoniae cannot grow in the presence of bile and is considered bile soluble, in contrast to the Group D Streptococci (i.e. Enterococci, S. bovis)which can grow in the presence of bile (Choice C).

Microscopic examination of a sputum sample from a 34-year-old male with fever and cough reveals Gram positive lancet-shaped cocci in pairs. These bacteria are likely to be: A. Catalase positive B. Optochin resistant C. Bile soluble D. Capable of complete hemolysis E. Bacitracin sensitive F. Able to grow in 6.5% NaCl

*The answer is A.* S.pneumoniae are alpha-haemolytic i.e. colonies are surrounded by a cloudy, colorless or greenish zone of partially lysed erythrocytes.

S.pneumoniae produces which of the following type of colonies? a) alpha-haemolytic b) beta-haemolytic c) non-haemolytic d) alpha-, beta-haemolytic

*The answer is C.* Clostridium tetrad produces disease by the production of a potent protein exotoxin, not by bacterial invasion of tissue. Even small amounts of tetanus toxin can be deadly. At first, the toxin binds to receptors on the presynaptic membranes of the motor neurons. From there the toxin migrates by the retrograde axonal transport system to the cell bodies of these neurons and next to the spinal cord and brain stem. Release of the inhibitory neurotransmitters glycine and gamma-aminobutyric acid (GABA) from these inhibitory neurons is blocked. The suppression of inhibitory nerve function results in an increased activation of nerves innervating muscles, causing muscle spasms, spastic paralysis and hyperreflexia. The muscle spasms involve both flexor and extensor muscles. Patients with tetanus have spastic muscle contractions, difficulty opening the jaw (known as lockjaw or "trismus"), a characteristic smile called "risus sardonicus" and contractions of back muscles, resulting in backward arching known as opisthotonos. Patients are extremely irritable, and develop tetanic seizures, brought about by violent, painful muscle contractions following minor stimuli such as a noise.

Neurotransmitter release from certain spinal cord inhibitory interneurons inhibits polysynaptic spread of impulses and insures proper functioning of agonist and antagonist skeletal muscles. Absence of agonist/antagonist muscle coordination and sustained impulse spread leading to spasm can result from inadequate release of: A. GABA due to C. difficile toxin B. Acetylcholine due to C. botulinum toxin C. Glycine due to C. tetani toxin D. Serotonin due to P. aeruginosa toxin E. Norepinephrine due to S. aureus toxin

*The answer is C.* Staphylococcus aureus is a common pathogen that should not be considered normal skin flora. Other staphylococci, especially Staphylococcus epidermidis, may contaminate blood cultures and can be differentiated from Staphylococcus aureus by the coagulase test. S. aureus is the only coagulase-positive staphylococcus. Bacitracin sensitivity differentiates Streptococcus pyogenes from the other beta-hemolytic streptococci, which are bacitracin resistant (choice A). The catalase test (choice B) is used to differentiate streptococci from staphylococci. Staphylococci produce catalase and can generate oxygen bubbles in hydrogen peroxide, whereas streptococci cannot. Novobiocin resistance (choice D) differentiates the coagulase-negative staphylococci into S. epidermidis (novobiocin sensitive) and S. saprophyticus (novobiocin resistant). Optochin resistance (choice E) differentiates the major pathogenic alpha-hemolytic streptococci. S. pneumoniae is optochin- and bile-sensitive whereas S. viridans is resistant to both optochin and bile.

One of three blood culture bottles drawn from a patient with unexplained fevers reveals gram-positive cocci growing in clusters. Which of the following tests would be most useful in determining whether this organism is a part of the normal skin flora? A. Bacitracin resistance B. Catalase C. Coagulase D. Novobiocin resistance E. Optochin resistance

*The answer is B.* The biological details and the type of diarrhea present indicate that this child has been infected by an organism of the Shigella species. Shigellosis is a hemorrhagic diarrheal disease caused by organisms of the genus Shigella. The most common species causing shigellosis in industrialized countries is Shigella sonnei, while the most common strain in developing nations is Shigella flexneri. Shigella is transmitted by the fecal-oral route and is never a component of the normal bacterial gut flora. Shigellosis is primarily a pediatric disease, but also shows a predilection for male homosexuals and adults in skilled nursing facilities. Mucosal invasion (Choice B) is the essential pathogenic mechanism for Shigella infection. Shigella invades the gastrointestinal mucosa specifically via M cells located in Peyer's patches. Shigella and EIEC both invade intestinal epithelial cells and release, respectively shiga toxin and shiga-like toxin. Shiga toxin causes cell destruction by inhibiting cellular protein synthesis. Shallow ulcers form where cells have sloughed oft; these ulcerations can be detected by colonoscopy. The shiga toxin is a classic bacterial AB exotoxin with a single A subunit bound to five B subunits. The A subunit inactivates the 60S ribosome of the host, thereby halting protein synthesis and causing cell death.

Stool cultures of a 4-year-old boy with self limited diarrhea and abdominal cramps grow non-lactose-fermenting gram negative rods on MacConkey's agar. Organisms do not produce gas on fermentation of glucose and do not produce H25 on TSI agar. Which of the following is the most important pathogenic mechanism for the infection caused by this bacterium? A. Exotoxin production B. Mucosal invasion C. Intestinal colonization D. Bacteremia E. Proliferation in lymph nodes


Related study sets

Ch 40: Musculoskeletal Care Modalities (4)

View Set

Med-Surg PrepU Chapter 51: Management of Patients with Female Reproductive Disorders

View Set

Conversion Factors for Converting Units of Volume to Liters

View Set

Starting Out with Python, 3e Ch 3

View Set